You are on page 1of 141

Problems for the course

Statistical Physics
(FYS3130)
Prepared by Yuri Galperin
Spring 2004
2
Contents
1 General Comments 5
2 Introduction to Thermodynamics 7
2.1 Additional Problems: Fluctuations . . . . . . . . . . . . . . . . . . . . . . . . . 28
2.2 Mini-tests . . . . . . . . . . . . . . . . . . . . . . . . . . . . . . . . . . . . . . 30
2.2.1 A . . . . . . . . . . . . . . . . . . . . . . . . . . . . . . . . . . . . . . 30
2.2.2 B . . . . . . . . . . . . . . . . . . . . . . . . . . . . . . . . . . . . . . 31
3 The Thermodynamics of Phase Transitions 33
3.1 Mini-tests . . . . . . . . . . . . . . . . . . . . . . . . . . . . . . . . . . . . . . 44
3.1.1 A . . . . . . . . . . . . . . . . . . . . . . . . . . . . . . . . . . . . . . 44
3.1.2 B . . . . . . . . . . . . . . . . . . . . . . . . . . . . . . . . . . . . . . 46
4 Elementary Probability Theory . . . 49
5 Stochastic dynamics ... 59
6 The Foundations of Statistical Mechanics 77
7 Equilibrium Statistical Mechanics 79
8 Tests and training 103
A Additional information 107
A.1 Thermodynamics . . . . . . . . . . . . . . . . . . . . . . . . . . . . . . . . . . 107
A.1.1 Thermodynamic potentials . . . . . . . . . . . . . . . . . . . . . . . . . 107
A.1.2 Variable transformation . . . . . . . . . . . . . . . . . . . . . . . . . . . 107
A.1.3 Derivatives from the equation of state . . . . . . . . . . . . . . . . . . . 107
A.2 Main distributions . . . . . . . . . . . . . . . . . . . . . . . . . . . . . . . . . . 109
A.3 The Dirac delta-function . . . . . . . . . . . . . . . . . . . . . . . . . . . . . . 110
A.4 Fourier Series and Transforms . . . . . . . . . . . . . . . . . . . . . . . . . . . 111
B Maple Printouts 115
3
4 CONTENTS
Chapter 1
General Comments
This year the course will be delivered along the lines of the book [1]. The problems will also be
selected from this book. It is crucially important for students to solve problems independently.
The problems will be placed on the course homepage. The same page will contain solutions
of the problems. So if a student is not able to solve the problem without assistance, then she/he
should come through the solution. In any case, student have to able to present solutions, obtained
independently, or with help of the course homepage.
FYS 3130 (former FYS 203) is a complicated course, which requires basic knowledge of
classical and quantum mechanics, electrodynamics, as well as basics of mathematics.
Using a simple test below please check if your knowledge is sufcient. Answers can be found
either in the Maple le.
A simple test in mathematics
Elementary functions
Problem 1.1: Function is given by the denition
f (x) = x
4
+ax
3
+bx
2
.
(a) Show that by a proper rescaling it can be expressed as
f (x) = a
4
F

() where F

() =
4
+
3
+
2
,
x/a, b/a
2
. Function F

() is very important in theory of phase transitions.


(b) Investigate F

().
How many extrema it has?
When it has only 1 minimum? When it has 2 minima? At what value of it has an
inection point?
5
6 CHAPTER 1. GENERAL COMMENTS
Plot F

() for this value of .


Problem 1.2: Logarithmic functions are very important in statistical physics. Check you mem-
ory by the following exercises:
Plot function
f
1
(x) = ln
1x
1+x
for [x[ 1.
Discuss properties of this function at [x[ > 1.
Plot function
f
2
(x) = ln(tanx).
Simplify the expression
e
4lnx
(x
2
+1)
2
+2x
2
+1.
Problem 1.3: Do you remember trigonometry? Test it!
Simplify
sin
2
x tan
2
x 1.
Calculate innite sums
C(, ) =

n=0
e
n
cosn, > 0.
S(, ) =

n=0
e
n
sinn, > 0.
Hint: take into account that cosx = Re
_
e
ix
_
, sinx = Im
_
e
ix
_
.
Basic integrals
Problem 1.4: Calculate interglars:

I
n
() =


0
x
n
e
x
dx, > 0.


dx
x
2
a
2
,

dx
x(1x)
.

G() =


0
e
x
2
/2
dx, > 0.
Chapter 2
Introduction to Thermodynamics
Quick access: 1 2 3 4 5 6 7 8 9 10 11 12 13 14 15 16 17 18 19 20 21 22
Problem 2.1: Test the following differentials for exactness. For those cases in which the dif-
ferential is exact, nd the function u(x, y).
(a) du
a
=
ydx
x
2
+y
2
+
xdy
x
2
+y
2
.
(b) du
b
= (y x
2
)dx +(x +y
2
)dy .
(c) du
c
= (2y
2
3x)dx 4xydy .
Solution 2.1:
(a) The differential is exact, u
a
(x, u) =arctan(x/y).
Here one point worth discussion. The function u
a
(x, u) = arctan(x/y) has a singularity
at x = 0, y = 0. As a result, any close path embedding this point contributes 2 to the vari-
ation of the quantity u(x, y). Consequently, this function cannot serve as a thermodynamic
potential if both positve and negative values of x and y have physical meaning.
(b) The differential is exact, u
b
(x, y) = yx +(y
3
x
3
)/3.
(c) The differential is not exact.
The function u(x, y) is reconstructed in the following way. For an exact differential, du =u
x
dx+
u
y
dy,
u
x
=
u(x, y)
x
, u
y
=
u(x, y)
y
.
If we introduce
u
1
(x, y) =

x
0
u
x
(, y)d,
7
8 CHAPTER 2. INTRODUCTION TO THERMODYNAMICS
then the difference f u(x, y) u
1
(x, y) is a function only of y. Consequently,
d f
dy
=
u(x, y)
y

u
1
(x, y)
y
= u
y
(x, y)

x
0
u
x
(x, y)
y
dx.
As a result,
f (y) =

y
0
du
y
(x, )

y
0

x
0
dd
u
x
(, )

.
Finally,
u(x, y) =

x
0
du
x
(, y) +

y
0
du
y
(x, )

y
0

x
0
dd
u
x
(, )

.
For calculation see the Maple le it1.mws.
Problem 2.2: Consider the two differentials
1. du
1
= (2xy +x
2
)dx +x
2
dy, and
2. du
2
= y(x 2y)dx x
2
dy.
For both differentials, nd the change u(x, y) between two points, (a, b) and (x, y). Compute the
change in two different ways:
(a) Integrate along the path (a, b) (x, b) (x, y),
(b) Integrate along the path (a, b) (a, y) (x, y).
Discuss the meaning of your results.
Solution 2.2: The calculations are shown in the Maple le. In the case (b) the results are
different because the differential is not exact.
Problem 2.3: Electromagnetic radiation in an evacuated vessel of volume V at equilibrium
with the walls at temperature T (black body radiation) behaves like a gas of photons having
internal energy U = aVT
4
and pressure P = (1/3)aT
4
, where a is Stefans constant.
(a) Plot the closed curve in the PV plane for a Carnot cycle using black body radiation.
(b) Derive explicitly the efciency of Carnot engine which uses black body radiation as its
working substance.
9

!
"
Vol ume
P
r
e
s
s
u
r
e
6
D
6
?
Figure 2.1: On the Carnot cycle with black-body radiation.
Solution 2.3: We will follow example shown in Exercise 2.2. Let us start with isotherms. At
the isotherms the pressure is V-independent, thus isotherms are horizontal, see Fig. 2.1 Along
the rst isothermal path,
Q
12
= U +PV = (4/3)aT
4
h
(V
2
V
1
) =
4
3
aT
4
h
V
1
_
V
2
V
1
1
_
. (2.1)
In a similar way,
Q
34
=
4
3
aT
4
c
V
4
_
1
V
3
V
4
_
. (2.2)
Now let us consider an adiabatic path. Along an adiabatic path,
dQ = 0 = dU +PdV = 4aVT
3
dT +aT
4
dV +(1/3)aT
4
dV = 4aVT
3
dT +(4/3)aT
4
dV .
Consequently,
dT
T
=
1
3
dV
V
VT
3
= const, PV
4/3
= const .
Let us start form the point 2 characterized by the values P
1
,V
2
and adiabatically expand the
gas to the point 3 characterized by the volume V
3
. We have V
2
T
3
h
= V
3
T
3
c
. In a similar way,
V
4
T
3
c
=V
1
T
3
h
. Combining these equalities, we get:
V
2
T
3
h
=V
3
T
3
c
, V
4
T
3
c
=V
1
T
3
h

V
2
V
3
=
V
1
V
4
=
_
T
c
T
h
_
3
(2.3)
Combining Eqs. (2.1), (2.2) and (2.3), we nd
W = Q
12
+Q
34
=
4
3
aT
4
h
V
1
_
V
3
V
4
1
__
1
T
c
T
h
_
.
10 CHAPTER 2. INTRODUCTION TO THERMODYNAMICS
Remember: along a closed path U = 0 and the total heat consumption is equal to mechanical
work.
On the
Q
12
=
4
3
aT
4
h
V
1
_
V
2
V
1
1
_
.
As a result,
=
W
Q
12
=
T
h
T
c
T
h
as it should be.
Problem 2.4: A Carnot engine uses a paramagnetic substance as its working substance. The
equation of state is
M =
nDH
T
where M is magnetization, H is the magnetic eld, n is the number of moles, D is a constant
determined by the type of substance, and T is is the temperature.
(a) show that the internal energy U, and therefore the heat capacity C
M
, can only depend on
the temperature and not the magnetization.
(b) Let us assume that C
M
=C = constant. Sketch a typical Carnot cycle in the MH plane.
(c) Compute the total heat absorbed and the total work done by the Carnot engine.
(d) Compute the efciency of the Carnot engine.
Solution 2.4:
(a) By denition [see Eq. [1](2.23)],
dU = T dS+HdM.
Thus at M = const the internal energy is independent of the magnetization.
(b) Since C = const, U
0
= Nc
M
T, where c
M
is the specic heat per one particle while N is the
number of particles. Introducing molar quantities we get U
0
= ncT.
A Carnot cycle is shown in Fig. 2.2. We have:
(c) For an isothermal process at T = T
c
,
Q
12
=

H
2
H
1
H(M)dM =
T
c
2nD
(M
2
1
M
2
2
).
In a similar way,
Q
34
=
T
h
2nD
(M
2
3
M
2
4
).
11
M
H
1
2
3
4
T
T
h
c
Figure 2.2: Sketch of the Carnot cycle.
The total work is then W = Q
12
+Q
34
, and the efciency is
=
W
Q
34
= 1+
T
c
T
h
M
2
1
M
2
2
M
2
3
M
2
4
.
Now let us discuss adiabatic paths. We have at each path,
0 = dQ = dU HdM = ncdT
MT
nD
dM.
Immediately we get
dT
T
=
1
n
2
cD
MdM.
Integrating this equality from point 2 to point 3 we obtain,
2n
2
cD ln
T
h
T
c
= M
3
3
M
2
2
.
In a similar way, integrating from 4 to 1 we obtain
2n
2
cD ln
T
c
T
h
= M
3
1
M
2
4
.
As a result,
M
2
3
M
2
2
= M
2
4
M
2
1
M
2
3
M
2
4
=(M
1
M
2
2
).
(d) Using this expression we obtain the efciency
=
Q
12
+Q
34
Q
34
=
T
h
T
c
T
h
.
12 CHAPTER 2. INTRODUCTION TO THERMODYNAMICS
Coming back to the item (c) we nd
W = Q
34
=
M
2
3
M
2
4
2nD
T
h
T
c
T
2
h
.
Problem 2.5: Find the efciency of the engine shown in Fig. 2.3 ([1] -Fig.2.18). Assume that
3
1
2
4
P
V
adi abati c
Figure 2.3: Sketch of the cycle.
the operating substance is an ideal monoatomic gas. Express your answer in terms of V
1
and V
2
.
(The processes 1 2 and 3 4 are adiabatic. The processes 4 1 and 2 3 occur at constant
volume).
v
Solution 2.5: Let us start with the processes at constant volume. The mechanical work during
theses processes does not take place. Consequently,
Q
23
= (3/2)nR(T
3
T
2
),
Q
41
= (3/2)nR(T
1
T
4
),
W = (3/2)nR(T
1
+T
3
T
2
T
4
).
The efciency is given by the expression
=
T
1
+T
3
T
2
T
4
T
1
T
4
= 1
T
2
T
3
T
1
T
4
. (2.4)
Now let us consider the adiabatic processes where TV
2/3
=const (monoatomic ideal gas!). Thus,
T
1
T
2
=
T
4
T
3
=
_
V
2
V
1
_
2/3
.
13
Substituting this expression into Eq. (2.4) we obtain:
= 1
1

= 1
_
V
1
V
2
_
2/3
.
Problem 2.6: One kilogram of water is compressed isothermally at 20

C from 1 atm to 20
atm.
(a) How much work is required?
(b) How much heat is rejected?
Assume that the average isothermal compressibility of water during this process is
T
= 0.5
10
4
(atm)
1
and the average thermal expansivity of water during this process is
P
= 210
4
(

C)
1
.
Solution 2.6: Since for an isothermal process dQ = T dS we have
Q = T

P
2
P
1
_
S
P
_
T
dP.
Using the Maxwell relation for the Gibbs free [see Eq. [1]-(2.112)] energy we obtain
_
S
P
_
T
=
_
V
T
_
P
=V
P
.
Thus
Q =T

P
2
P
1
V(P)
T
(P)dP
Now let us assume that
T
it P-independent, and
V(P) =V
0
[1
T
(PP
0
)] = (M/)[1
T
(PP
0
)] .
As a result,
Q =(M/)T
T
(P
2
P
1
)[1
T
(PP
0
)/2] .
We see that since the compressibility of water is very low one can neglect the correction due to
change in the volume and assume V M/. The mechanical work is
W =

P
2
P
1
PdV =

P
2
P
1
P
_
V
P
_
T
dP = (M/2)
T
(P
2
2
P
2
1
).
14 CHAPTER 2. INTRODUCTION TO THERMODYNAMICS
2J
0
2L
0
J
0
L
0
a
b
c
Figure 2.4:
Problem 2.7: Compute the efciency of the heat engine shown in Fig. 2.4 (Fig. [1]-2.19). The
engine uses a rubber band whose equation of state is
J = LT , (2.5)
where is a constant, J is the tension, L is the length per unit mass, and T is the temperature in
Kelvins. The specic heat (heat capacity per unit mass) is a constant, c
L
= c.
Solution 2.7: From Fig. 2.4 we see that the path a b is isothermal. Indeed, since J L, it
follow from Eq. ( 2.5) that T = const. Then, from the same equation we get,
T
a
= T
b
= J
0
/L
0
, T
c
= J
0
/2L
0
= T
a
/2.
As a result,
Q
ba
= MT
a

2L
0
L
0
LdL = (3/2)ML
2
0
T
a
= (3/2)MJ
0
L
0
,
Q
ac
= Mc(T
c
T
a
) =(1/2)McT
a
,
Q
cb
= Mc(T
b
T
c
) MJ
0
L
0
= (1/2)McT
a
MJ
0
L
0
.
Hence, the total work is
W = Q
ba
+Q
ac
+Q
cb
= (1/2)MJ
0
L
0
and
=
W
Q
ba
=
1
3
.
This result is also clear from geometrical point of view.
15
Problem 2.8: Experimentally one nds that for a rubber band
_
J
L
_
T
=
aT
L
0
_
1+2
_
L
0
L
_
3
_
,
_
J
T
_
L
=
aL
L
0
_
1
_
L
0
L
_
3
_
,
where J is the tension, a = 1.010
3
dyne/K, and L
0
= 0.5 m is the length of the band when no
tension is applied. The mass of the rubber band is held xed.
(a) Compute (L/T)
J
and discuss its physical meaning.
(b) Find the equation of state and show that dJ is an exact differential.
(c) Assume that heat capacity at constant length is C
L
= 1.0 J/K. Find the work necessary to
stretch the band reversibly and adiabatically to a length 1 m. Assume that when no tension
is applied, the temperature of the band is T = 290 K. What is the change in temperature?
Solution 2.8:
(a) We have
_
L
T
_
J

_
T
J
_
L

_
J
L
_
T
=1.
Consequently,
_
L
T
_
J
=
__
T
J
_
L

_
J
L
_
T
_
1
=
_
J
T
_
L
_
J
L
_
T
=
L
T
1
_
L
0
L
_
3
1+2
_
L
0
L
_
3
.
Physical meaning -

J
=
1
L
_
L
T
_
J
is the thermal expansion coefcient at given tension.
(b) The equation of state has the form
J =
aTL
L
0
_
1
_
L
0
L
_
3
_
.
The proof of the exactness is straightforward.
16 CHAPTER 2. INTRODUCTION TO THERMODYNAMICS
(c) Consider adiabatic expansion of the band,
0 = dQ =C
L
dT +J(L, T)dL.
Consequently,
dT
dL
=
J(L, T)
C
L
=
aTL
C
L
L
0
_
1
_
L
0
L
_
3
_
.
Measuring length in units of L
0
as L = L
0
and introducing aL
0
/C
L
we obtain the
following differential equation
dT/T =(1
3
)d.
Its solution is
ln
T
f
T
0
=

2
_
L
f
L
0
_
2
_
1+2
_
L
0
L
f
_
3
_
.
Here L
f
and T
f
are nal values of the length and temperature, respectively. This is the
equation for adiabatic process which provides the change in the temperature. The mechan-
ical work is then
W =C
L
(T
f
T
0
).
Problem 2.9: Blackbody radiation in a box of volume V and at temperature T has internal
energy U = aVT
4
and pressure P = (1/3)aT
4
, where a is the Stefan-Boltzmann constant.
(a) What is the fundamental equation for the blackbody radiation (the entropy)?
(b) Compute the chemical potential.
Solution 2.9:
(a) Let us rst nd the Helmholtz free energy. From the relation P =
_
A
V
_
T
we get
A =PV =
1
3
aVT
4
=
1
3
U .
Consequently,
S =
_
A
T
_
V
=
4
3
aVT
3
=
4
3
U
T
.
As a result,
_
S
U
_
V
=
(S,V)
(U,V)
=
(S,V)/(T,V)
(U,V)/(T,V)
=
(S/T)
V
(U/T)
V
=
1
T
.
(b) Since A is N-independent, = 0.
17
Problem 2.10: Two vessels, insulated from the outside world, one of volume V
1
and the other
of volume V
2
, contain equal numbers N of the same ideal gas. The gas in each vessel is originally
at temperature T
i
. The vessels are then connected and allowed to reach equilibrium in such a way
that the combined vessel is also insulated fromthe outside world. The nal volume is V =V
1
+V
2
.
What is the maximum work, W
f ree
, that can be obtained by connecting these insulated vessels?
Express your answer in terms of T
i
, V
1
, V
2
, and N.
Solution 2.10: The Gibbs free energy of an ideal gas is given by the equation
G = NkT lnP+N(T)
where (T) is some function of the gas excitation spectrum.
1
Consequently,
S =
_
G
T
_
P
=NklnPN
/
(T).
Before the vessels are connected,
S
i
=Nkln(P
1
P
2
) 2N
/
(T).
After the vessels are connected the temperature remains the same, as it follows from the conser-
vation law, the entropy being
S
f
=2NklnP2N
/
(T).
Consequently, S =Nkln(P
2
/P
1
P
2
). On the other hand,
1
P
=
V
1
+V
2
2NkT
i
,
1
P
i
=
V
i
NkT
i

P
2
P
1
P
2
=
4V
1
V
2
(V
1
+V
2
)
2
.
1
For an ideal gas, the energy of the particle can be written as a sum of the kinetic energy,
p
= p
2
/2m, and the
energy of internal excitations,

(characterized by some quantum numbers ),

p
=
p
+

.
As we will see later, the free energy of the ideal gas can be constructed as
A =
kT
N!
ln
_

p
e

p
/kT
_
N
NkT ln
_
eV
N
_
mkT
2 h
2
_
3/2

/kT
_
= NT ln(eV/N) +N f (T),
f (T) = kT ln
_
_
mkT
2 h
2
_
3/2

/kT
_
.
The Gibbs free energy is then
G = A+PV = NkT lnP+N(T), (T) f (T) kT lnkT .
18 CHAPTER 2. INTRODUCTION TO THERMODYNAMICS
As a result, the maximum work is
W
f ree
= T
i
S = NkT
i
ln
(V
1
+V
2
)
2
4V
1
V
2
.
Problem 2.11: For a low-density gas the virial expansion can be terminated at rst order in the
density and the equation of state is
P =
NkT
V
_
1+
N
V
B
2
(T)
_
,
where B
2
(T) is the second virial coefcent. The heat capacity will have corrections to its ideal
gas value. We can write it in the form
C
V,N
=
3
2
Nk
N
2
k
V
f (T).
(a) Find the form that f (T) must have in order for the two equations to be thermodynamically
consistent.
(b) Find C
P,N
.
(c) Find the entropy and internal energy.
Solution 2.11:
(c) The equation of state under consideration can be obtained from the Helmholtz free energy
2
A = A
ideal
+kTB
2
(T)
N
2
V
.
Then
S =
_
A
T
_
V
= S
ideal
+S, S k
N
2
V
_
B
2
(T) +TB
/
2
(T)

.
Since we know both entropy and Helmholtz free energy, we nd the internal energy as
U = A+TS =U
ideal
+kTB
2
(T)
N
2
V
kT
N
2
V
_
B
2
(T) +TB
/
2
(T)

= U
ideal
+U , U kT
2
B
/
2
(T)
N
2
V
.
2
Remember that P =
_
A
V
_
T
.
19
(a) As a result,
C
V
= T
_
S
T
_
V
=
_
U
T
_
V
=C
ideal
V
+C
V
, C
V
kT
N
2
V
_
2B
/
2
(T) +TB
//
2
(T)

.
We nd in this way,
f (T) = 2TB
/
2
(T) +T
2
B
//
2
(T).
(b) Let us express the equation of state as V(P, T). Since the density is assumed to be small in
the correction one can use equation for the ideal gas to nd the the volume. We have,
V =
NkT
P
+NB
2
(T).
Consequently, the entropy can be expressed as
S = S
ideal
+S
1
, S
1
kN
B
2
+TB
/
2
kT/P+B
2
.
Now
C
P
= C
ideal
P
+T
_
S
1
T
_
P
= kN
(B
2
+TB
/
2
)
/
(kT/P+B
2
) (k/P+B
/
2
)(B
2
+TB
/
2
)
(kT/P+B
2
)
2
= C
ideal
P
k
N
2
V
_
T(2B
/
2
+TB
//
2
) (B
2
+TB
/
2
)

= C
ideal
P
+C
V
+k
N
2
V
(B
2
+TB
/
2
).
Here in all corrections we used equation of state for an ideal gas. As a result,
C
P
C
V
= (C
P
C
V
)
ideal
+k
N
2
V
(B
2
+TB
/
2
).
Problem 2.12: Prove that
C
Y,N
=
_
H
T
_
Y,N
and
_
H
Y
_
T,N
= T
_
X
T
_
Y,N
X .
20 CHAPTER 2. INTRODUCTION TO THERMODYNAMICS
Solution 2.12: Let us rst recall denitions for X, Eq. ([1]-2.66), U = ST +YX +
J
j
/
j
dN
j
.
The enthalpy is dened as
H =U XY = ST +

/
j
dN
j
.
Since dU = T dS+Y dX we get
dH = T dSX dY .
Consequently,
C
Y,N
= T
_
S
T
_
Y,N
=
_
H
T
_
Y,N
.
To prove the second relation we do the following
_
H
Y
_
T,N
=
_
H
Y
_
S,N
+
_
H
S
_
Y,N
_
S
Y
_
T
.
Now,
_
H
Y
_
S,N
=X ,
_
H
S
_
Y,N
= T .
Now we have to use the Maxwell relation, which emerges for the Gibbs free energy G=HTS.
From
dG =SdT X dY
we get
_
S
Y
_
T
=
_
X
T
_
Y,N
.
Thus we obtain the desired result.
Problem 2.13: Compute the entropy, enthalpy, Helmholtz free energy, and Gibbs free energy
for a paramagnetic substance and write them explicitly in terms of their natural variables if pos-
sible. Assume that mechanical equation of state is m = (D/T)H and the the molar heat capacity
at constant magnetization is c
m
= c, where m is the molar magnetization, H is the magnetic eld,
D is a constant, c is a constant, and T is the temperature.
Solution 2.13: Let us start with the internal energy u(T, m) per one mole. We have the mag-
netic contribution u
mag
=

m
0
H(m)dm. Since H(m) = (T/D)m we get u
mag
= (T/2D)m
2
. The
thermal contribution is cT. As a result,
u(T, m) = T(c +m
2
/2D), T(u, m) =
u
c +m
2
/2D
.
The molar entropy s is then derived from the denition
_
s
u
_
m
=
1
T
=
c +m
2
/2D
u
s(u, m) = (c +m
2
/2D)ln(u/u
0
).
21
Here u
0
is a constant. As a result, in natural variables
u(s, m) = u
0
exp
_
s
c +m
2
/2D
_
.
To nd other thermodynamic potentials we need s(T, m). We can rewrite the above expression
for the entropy as
s(T, m) = (c +m
2
/2D)ln
T(c +m
2
/2D)
u
0
.
In particular, the Helmholtz free energy is
a(T, m) = uTs = T(c +m
2
/2D)
_
1ln
T(c +m
2
/2D)
u
0
_
.
To get enthalpy we have to subtract from u the quantity Hm= (D/T)H
2
and to express m through
H as m = (D/T)H. As a result, we obtain:
h(T, H) = uHm = T(c +DH
2
/2T
2
) (D/T)H
2
= T(c DH
2
/2T
2
).
Finally, Gibbs free energy is g = aHm, which has to be expressed through T and H. We get
g = aHm = T(c +DH
2
/2T
2
)
_
1ln
T(c +DH
2
/2T
2
)
u
0
_
(D/T)H
2
= T(c DH
2
/2T
2
) T(c +DH
2
/2T
2
)ln
T(c +DH
2
/2T
2
)
u
0
.
Problem 2.14: Compute the Helmholtz free energy for a van der Waals gas. The equation of
state is
_
P+
n
2
V
2
_
(V nb) = nRT ,
where and b are constants which depend on the type of gas and n is the number of moles.
Assume that heat capacity is C
V,n
= (3/2)nR.
Is this a reasonable choice for the heat capacity? Should it depend on volume?
Solution 2.14: Let us express pressure through the volume,
P =
nRT
V nb

n
2
V
2
.
Since P =A/V we obtain
A =

P(V)dV =nRT ln(V nb) (n


2
/V) +A(T).
22 CHAPTER 2. INTRODUCTION TO THERMODYNAMICS
Here A(T) is the integration constant, which can be found from the given specic heat. Indeed,
S =

T
C
V,n
dT
/
T
/
= (3/2)nRln(T/T
0
).
Consequently,
A(T) =

T
S(T
/
)dT
/
= (3/2)nRT[1ln(T/T
0
)] .
Here we omit temperature-independent constant.
The suggestion regarding specic heat is OK since the difference between the entropies of
van der Waals gas and the ideal gas is temperature independent. (Check!)
Problem 2.15: Prove that
(a)
T
(C
P
C
V
) = TV
2
P
(b) C
P
/C
V
=
T
/
S
.
Solution 2.15: We use the method of Jacobians:
(a)
C
V
= T (S/T)
V
= T(S,V)/(T,V) =
(S,V)/(T, P)
(T,V)/(T, P)
= T
(S/T)
P
(V/P)
T
(S/P)
T
(V/T)
P
(V/P)
T
= C
P
T
(S/P)
T
(V/T)
P
(V/P)
T
.
Now, from the Maxwell relations (S/P)
T
=(V/T)
P
. Thus,
C
P
C
V
=T
[(V/T)
P
]
2
(V/P)
T
= TV

2
P

T
.
The rst relation follows from this in a straightforward way from denitions.
(b) Let us rst calculate the adiabatic compressibility (V/P)
S
as
_
V
P
_
S
=
(V, S)
(P, S)
=
(V, S)/(V, T)
(P, S)/(P, T)

(V, T)
(P, T)
=
(S/T)
V
(S/T)
P

_
V
P
_
T
.
Consequently,
C
P
C
V
=
(V/P)
T
(V/P)
S
=

T

S
.
23
Problem 2.16: Show that
Tds = c
x
(T/Y)
x
dY +c
Y
(T/x)
Y
dx,
where x = X/n is the amount of extensive variable, X, per mole, c
x
is the heat capacity per mole
at constant x, and c
Y
is the heat capacity per mole at constant Y.
Solution 2.16: Let us substitute the denitions
c
x
= T (S/T)
x
, c
Y
= T (S/T)
Y
.
Now the combination c
x
(T/Y)
x
dY +c
Y
(T/x)
Y
dx can be rewritten as
T (S/T)
x
(T/Y)
x
dY +T (S/T)
Y
(T/x)
Y
dx
= T (S/Y)
x
dY +T (S/x)
Y
dx = T ds.
Problem 2.17: Compute the molar heat capacity c
P
, the compressibilities,
T
and
S
, and
the thermal expansivity
P
of a monoatomic van der Waals gas. Start from the fact that the
mechanical equation of state is
P =
RT
v b


v
2
.
and the molar heat capacity is c
v
= 3R/2, where v =V/n is the molar volume.
Solution 2.17: Let us start with ther specic heat. Using the method similar to the Problem
2.15 we can derive the relation
c
P
c
v
=T
[(P/T)
v
]
2
(P/v)
T
=
R
12(v b)
2
/RTv
3
.
Now let us compute the compressibility

T
=
1
v
_
v
P
_
T
=
1
v
_
P
v
_
1
T
=
(v b)
2
vRT
1
12(v b)
2
/RTv
3
.
Given c
v
, other quantities can be calculated using results of the Problem 2.15.
Problem 2.18: Compute the heat capacity at constant magnetic eld C
H,n
, the susceptibilities

T,n
and
S,n
, and the thermal expansivity
H,n
for a magnetic system, given that the mechanical
equation of state is M = nDH/T and the heat capacity C
M,n
= nc, where M is the magnetization,
H is the magnetic eld, n is the number of moles, c is the molar heat capacity, and T is the
temperature.
24 CHAPTER 2. INTRODUCTION TO THERMODYNAMICS
Solution 2.18: Let us start with susceptibilities. By denition,

T,n
=
_
M
H
_
T,n
=
nD
T
.
Now,
_
M
H
_
S
=
(M, S)
(H, S)
=
(M, S)/(M, T)
(H, S)/(H, T)

(M, T))
(H, T)
=
T,n
C
M,n
C
H,n
.
Thus we have found one relation between susceptibilities and heat capacities,

S,n

T,n
=
C
M,n
C
H,n
.
Now let us nd C
H,n
. At constant H, M becomes dependent only on temperature. Then,
dM =
dM
dT
dT =
nDH
T
2
dT .
Consequently, the contribution to the internal energy is
dU =HdM =
nDH
2
T
2
dT .
As a result,
C
H,n
C
M,n
=
nDH
2
T
2
.
Given C
H,n
= nc we easily compute C
H,n
and
S,n
.
According to Eq. (R2.149),
H
is dened as

H
=
_
M
T
_
H
=
nDH
T
2
.
Problem 2.19: A material is found to have a thermal expansivity
P
= v
1
(R/P+a/RT
2
) and
an isothermal compressibility
T
= v
1
[T f (P) +b/P]. Here v =V/n is the molar volume.
(a) Find f (P).
(b) Find the equation of state.
(c) Under what condition this materials is stable?
25
Solution 2.19:
(a) By denition, we have
v
T
=
R
P
+
a
RT
2
.
v
P
= T f (P)
b
P
.
To make dv an exact differential we need:

R
P
2
=f (p).
Thus
f (p) = R/P
2
.
(b) We can reconstruct the equation of state as:
v =

P
v
P
dP =

P
dP
_

RT
P
2

b
P
_
=
RT
P
blnP+g(T).
Here g(T) is some function of the temperature. Now,
v
T
=
R
P
+g
/
(T)
R
P
+
a
RT
2
.
Thus g(T) =a/RT +const. As a result, we can express the equation of state as
v v
0
=
RT
P
+bln
P
0
P

a
RT
.
(c) Since the compressibility must be positive, we have the stability condition
T f (P) +
b
P
> 0
TR
P
2
+
b
P
> 0.
Consequently, the stability condition is
P/T < R/b.
Problem 2.20: Compute the efciency of the reversible two heat engines in Fig. 2.5 (R2.20).
Which engine is the most effective? (Note that these are not Carnot cycles. The efciency of a
heat engineis = W
total
/Q
absorbed
.
26 CHAPTER 2. INTRODUCTION TO THERMODYNAMICS
6
6

5
6
6

5
=
=
>
>
?
?
=
>
Figure 2.5:
Solution 2.20: Since dQ = T dS, we immediately get for any closed path in the T S plane:
W
total
=

T dS.
This is just the area of the triangle,
W
total
= (1/2)(T
2
T
1
)(S
2
S
1
).
The heat absorbed in the case (a) is
Q
absorbed
= T
2
(S
2
S
1
).
Thus,

a
=
T
2
T
1
2T
2
.
In the case (b), it easy to show that
Q
absorbed
= (1/2)(T
2
+T
1
)(S
2
S
1
).
Thus

b
=
T
2
T
1
T
2
+T
1
>
a
.
Problem 2.21: It is found for a gas that
T
= Tv f (P) and
P
= Rv/P+Av/T
2
, where T is
the temperature, v is the molar volume, P is the pressure, A is a constant, and f (P) is unknown
function.
(a) What is f (P)?
(b) Find v = v(P, T).
27
Solution 2.21: The solution is similar to the problem 2.19. We have:
v
T
= v
2
_
R
P
+
A
T
2
_
,
v
P
= v
2
T f (P).
Let us introduce (P, T) [v(P, T)]
1
. We get,

T
=
R
P

A
T
2
,

P
= T f (P).
Again, from the Maxwell relation we get
R
P
2
= f (p). Then we can express as
=

P
dP = TR

P
dP
P
2
=
RT
P
+g(T).
Then, g
/
(T) =A/T
2
, or g(T) = A/T +const. As a result,
=
0
+
A
T

RT
P
.
Consequently,
v(P, T) =
1

0
+A/T RT/P
.
Problem 2.22: A monomolecular liquid at volume V
L
and pressure P
L
is separated from a gas
of the same substance by a rigid wall which is permeable to the molecules, but does not allow
liquid to pass.The volume of the gas is held xed at V
G
, but the volume of the liquid cam be
varied by moving a piston. If the pressure of the liquid is increased by pushing in on the piston,
by how much does the pressure of the gas change? [Assume the liquid in incompressible (its
molar volume is independent of the pressure) and describe the gas by the ideal gas equation of
state. The entire process occurs at xed temperature T].
Solution 2.22: Let us consider the part of the system, which contains both liquid and gas
particles. In this part the chemical potentials must be equal,
L
=
G
. On the other hand, the
chemical potentials and pressures of gas in both parts must be equal. Thus we arrive at the
equation,

L
(P
L
, T) =
G
(P
G
, T).
If one changes the pressure of liquid by P
L
, then
_

L
P
L
_
T
P
L
=
_

G
P
G
_
T
P
G
.
28 CHAPTER 2. INTRODUCTION TO THERMODYNAMICS
For an ideal gas, (/P)
T
= kT/P
G
=V
G
/N
G
v
L
. the quantity v
L
has a physical meaning of
the volume per particle. For a liquid, the relation is the same,
_

P
_
T
=

P
_
G
N
_
P,T
=

N
_
G
P
_
N,T
=
_
V
N
_
T
= v
L
.
The last relation is a consequence of incompressible character of the liquid. As a result,
P
G
P
L
=
v
L
v
G
.
2.1 Additional Problems: Fluctuations
Quick access: 23 24 25 26 27
Problem 2.23: Find the mean square uctuation of the internal energy (using V and T as inde-
pendent variables). What is the mean square uctuation of the internal energy for a monoatomic
ideal gas?
Solution 2.23: We have
U =
_
U
V
_
T
V +
_
U
T
_
V
T =
_
T
_
P
T
_
V
P
_
V +C
V
T .
Here we use Maxwell relation which can be obtained from Helmholtz free energy. Squaring and
averaging we obtain (note that V T) = 0)
(U)
2
) =
_
T
_
P
T
_
V
P
_
2
(V)
2
) +C
2
V
(T)
2
).
Now,
(T)
2
) = kT
2
/C
V
, (V)
2
) =kT
_
V
P
_
T
.
Thus
(U)
2
) =kT
_
T
_
P
T
_
V
P
_
2
_
V
P
_
T
+C
V
kT
2
.
For the ideal gas,
P = NkT/V , C
V
= (3/2)Nk.
Thus,
(U)
2
) = (3/2)N(kT)
2
.
2.1. ADDITIONAL PROBLEMS: FLUCTUATIONS 29
Problem 2.24: Find T P) (with variables V and T) in general case and for a monoatomic
ideal gas.
Solution 2.24: We have
P =
_
P
V
_
T
V +
_
P
T
_
V
T .
Squaring and averaging we obtain (note that V T) = 0)
T P) =
_
P
T
_
V
(T)
2
) =
_
kT
2
C
V
_ _
P
T
_
V
.
For an ideal gas,
T P) =
2
3
kT
2
V
.
Problem 2.25: Find V P) with variable (V and T).
Solution 2.25: Using results of the previous problem we get
V P) =
_
P
V
_
T
(V)
2
) =kT .
Problem 2.26: Using the same method show that
SV) = kT
_
V
T
_
P
, ST) = kT .
Solution 2.26: Starightforward.
Problem 2.27: Find a mean square uctuation deviation of a simple pendulum with the length
suspended vertically.
Solution 2.27: Let m be the pendulum mass, and is the angle of deviation from the vertical.
The minimal work is just the mechanical work done against the gravity force. For small ,
W
min
= mg (1cos) mg
2
/2.
Thus,

2
) = kT/mg.
30 CHAPTER 2. INTRODUCTION TO THERMODYNAMICS
2.2 Mini-tests
2.2.1 A
The Helmholtz free energy of the gas is given by the expression
A = N
0
NkT ln(eV/N) NcT lnT NT .
Here e =2.718. . . is the base of natural logarithms, N is the number of particles, V is the volume,
T is the temperature in the energy units, while
0
, c and are constants.
(a) Find the entropy as function of V and T.
(b) Find internal energy U as a function of the temperature T and number of particles N.
(c) Show that c is the heat capacity per particle at given volume, c
v
.
(d) Find equation of state.
(e) Find Gibbs free energy, enthalpy. Find the entropy as a function of P and T.
(f) Using these expression nd the heat capacity at constant pressure, c
P
.
(g) Show that for an adiabatic process
T

P
1
= constant, where = c
P
/c
v
.
Solution
By denition,
S =
_
A
T
_
V,N
= Nln
eV
N
+Nc(1+lnT) +NT .
Now,
U = A+TS = N
0
+NcT ,
P =
_
A
V
_
T,N
=
NT
V
,
G = A+PV = A+NT = N
0
NT ln
V
N
NcTlT +NT
= N
0
+NT lnPNT(1+c)lnT NT ,
W = U +PV =U +NT = N
0
+N(c +1)T ,
S = NlnP+N(1+c)lnT +N(+1+c),
c
P
= T
_
S
T
_
P,N
= 1+c.
2.2. MINI-TESTS 31
Immediately, to keep entropy constant we get
NlnP+Nc
P
lnT = const T
c
P
/P = const .
Since c
P
c
V
= c
P
c = 1, we obtain
T
c
P
P
c
P
c
V
= const , T

P
1
= const .
2.2.2 B
Problem 1
Discuss entropy variation for
(a) adiabatic process,
(b) isothermic process,
(c) isochoric process,
(d) isobaric process.
Problem 2
(a) Discuss the difference between Gibbs and Helmholtz free energy.
(b) Prove the relation
U =T
2
_

T
A
T
_
.
(c) A body with constant specic heat C
V
is heated under constant volume from T
1
to T
2
. How
much entropy it gains?
(d) Discuss the heating if the same body is in contact with a thermostat at T
2
. In the last case the
heating is irreversible. Show that the total entropy change is positive.
(e) Two similar bodies with temperatures T
1
and T
2
brought into contact. Find the nal tempera-
ture and the change in entropy.
Solution
Problem 1
(a) Constant
(b)
S = Q/T .
(c)
S =

T
2
T
1
C
V
(T)
T
dT =C
V
ln
T
2
T
1
.
32 CHAPTER 2. INTRODUCTION TO THERMODYNAMICS
(d)
S =

T
2
T
1
C
P
(T)
T
dT =C
P
ln
T
2
T
1
.
Problem 2
Mechanical work under isothermic process is given by
dW = dU dQ = dU T dS = d(U TS).
The function of state
A =U TS
is called Helmholtz free energy. We have
dA = dU T dSSdT = (T dSPdV) T dSSdT =SdT PdV.
Thus,
S =
_
A
T
_
V
, P =
_
A
V
_
T
.
Thus, A is the thermodynamic potential with respect to V and T.
The thermodynamic potential with respect to P and T is called Gibbs free energy. We have
G =U TSPV dG =SdT +V dP.
(b) Substituting
S =
_
A
T
_
V
into denition of A we get the result.
(c) We have
S =

T
2
T
1
C
V
T
dT =C
V
ln
_
T
2
T
1
_
.
(d) Use 2nd law of thermodynamics
(e) Energy conservation law yields
C
V
(T
2
T
B
) =C
V
(T
B
T
1
) T
B
= (T
1
+T
2
)/2.
for the entropy change we have
S =C
V
ln
T
B
T
1
+C
V
ln
T
B
T
2
= 2C
V
ln
_
T
1
+T
2
2

T
1
T
2
_
0.
Chapter 3
The Thermodynamics of Phase Transitions
Quick access: 1 2 3 4 5 6 7 8 9 10 11 12 13
Problem 3.1: A condensible vapor has a molar entropy
s = s
0
+Rln
_
C(v b)
_
u+
a
v
_
5/2
_
,
where c and s
0
are constants.
(a) Compute equation of state.
(b) Compute the molar heat capacities, c
c
and c
P
.
(c) Compute the latent heat between liquid and vapor phases temperature T in terms of the
temperature T, the gas constant R and gas molar volumes v
l
and v
g
. How can you nd
explicit values of v
l
and v
g
if you need to?
Solution 3.1:
a) Let us rst nd the temperature as function of u and s. We have
1
T
=
_
s
u
_
v
=
5R
2
_
u+
a
v
_
1
.
Thus,
s = s
0
+Rln
_
C(5RT/2)
5/2
(v b)
_
,
u =
5
2
RT
a
v
,
a = uTs =
5
2
RT
a
v
T
_
s
0
+Rln
_
C(v b)(5RT/2)
5/2
__
.
33
34 CHAPTER 3. THE THERMODYNAMICS OF PHASE TRANSITIONS
As a result,
P =
_
a
v
_
T
=
RT
v b

a
v
2
.
This is the van der Waals equation.
(b) We have,
c
v
= T
_
s
T
_
v
=
5
2
RT .
The result for c
P
can be obtained in the same way as in the problem 2.17.
(c) The latent heat q is just
q = T(s
g
s
l
) = RT ln
v
g
b
v
l
b
.
Since the pressure should be constant along the equilibrium line, P
l
= P
G
, we have
RT
v
g
b

a
v
2
g
=
RT
v
l
b

a
v
2
l
.
Another equation is the equality of chemical potentials,
l
=
g
. We know that =
(a/n)
T,V
, so everything could be done. Another way is to plot the isotherm and nd
the volumes using the Maxwell rule.
Problem 3.2: Find the coefcient of thermal expansion,
coex
= v
1
(v/T)
coex
, for a gas
maintained in equilibrium with its liquid phase. Find an approximate explicit expression for

coex
, using the ideal gas equation of state. Discuss its behavior.
Solution 3.2: It is implicitly assumed that the total volume of the system is kept constant. We
have,
v
T
=
_
v
T
_
P
+
_
v
P
_
T
dP
dT
.
For an ideal gas, v = RT/P. Thus
_
v
T
_
P
= R/P,
_
v
P
_
T
=RT/P
2
.
Consequently,

coex
=
1
v
_
v
T
_
coex
=
1
T
_
1
T
P
_
P
T
_
coex
_
.
According to the Clapeyron-Clausius formula,
_
P
T
_
coex
=
q
T(v
g
v
l
)

q
Tv
g
,
35
where q = (h)
lg
, we can rewrite this expression as

coex

1
T
_
1
q
RT
_
.
The coefcient of thermal expansion is less because with the increase of the temperature under
given pressure the heat is extracted from the system.
Problem 3.3: Prove that the slope of the sublimation curve of a pure substance at the triple
point must be greater than that of the vaporization curve at the triple point.
Solution 3.3: The triple point is dened by the following equations for the 2 phases:
P
1
= P
2
= P
3
, T
1
= T
2
= T
3
,
1
=
2
=
3
.
The denitions of the sublimation and vaporization curves are given in Fig. 3.4 of the text-
book [1]. Consequently, the slopes of the vaporization and sublimation curves are given by the
relations
_
P
T
_
lg
=
s
g
s
l
v
g
v
l

s
g
s
l
v
g
,
_
P
T
_
sg
=
s
g
s
s
v
g
v
s

s
g
s
s
v
g
.
Since solid state is more ordered, s
s
< s
l
, and
_
P
T
_
lg

_
P
T
_
sg
=
s
s
s
l
v
g
< 0.
Problem 3.4: Consider a monoatomic uid along its uid-gas coexistence curve. Compute the
rate of change of chemical potential along the coexistence curve, (/T)
coex
, where is the
chemical potential and T is the temperature. Express your answer in terms of s
l
, v
l
and s
g
, v
g
which are the molar entropy and molar volume of the liquid and gas, respectively.
Solution 3.4: We have (P, T) =
l
(P, T) =
g
(P, T). Thus,
d
dT
=
_

g
T
_
P
+
_

g
P
_
T
dP
dT
.
Consequently,
_
d
dT
_
coex
=s
g
+v
g
s
g
s
l
v
g
v
l
=
s
g
v
l
v
g
s
l
v
g
v
l
.
36 CHAPTER 3. THE THERMODYNAMICS OF PHASE TRANSITIONS
Problem 3.5: A system in its solid phase has a Helmholtz free energy per mole, a
s
= B/Tv
3
,
and in its liquid phase it has a Helmholtz free energy per mole, a
l
= A/Tv
2
, where A and B are
constants, v is the volume per mole, and T is the temperature.
(a) Compute the Gibbs free energy density of the liquid and solid phases.
(b) How are the molar volumes, v, of the liquid and solid related at the liquid-solid phase
transition?
(c) What is the slope of the coexistence curve in the PT plane?
Solution 3.5:
(a) By denition, g = a+Pv, and P =(a/v)
T
. Thus,
g = av(a/v)
T
, g
s
= 4B/Tv
3
s
, g
l
= 3A/Tv
2
l
.
(b) Since P =(a/v)
T
we have,
P
s
= 3B/Tv
4
s
, P
l
= 2A/Tv
3
l
.
Since at the phase transition P
s
= P
l
= P, we obtain
v
3
l
v
4
s
=
2A
3B
.
(c) Now we can express the Gibbs free energies per mole in terms of P,
g
s
=
4
3
(3B)
1/4
P
3/4
T
1/4
, g
l
=
3
2
(2A)
1/3
P
2/3
T
1/3
.
Since at the transition point g
l
= g
s
, we have
P
T
=
_
3
7/4
2
8/3
A
1/3
B
1/4
_
12
=
3
21
2
24
A
4
B
3
.
Problem 3.6: Deduce the Maxwell construction using stability properties of the Helmholtz
free energy rather than the Gibbs free energy.
Solution 3.6: Since the system in the equilibrium, the maximum work extracted during the
process of the phase transition is AP
0
V. Let us look at Fig. 3.1.
37
P
V
a
b
c
v
l
v
g
P
0
Figure 3.1:
We see that P
0
V = P
0
(V
g
V
l
) is just the area of the dashed rectangle. To nd A let is follow
isotherm. Since T = const,
A =

c
a
PdV ,
which is just the area below the isotherm abc. Subtracting areas we reconstruct the Maxwell
rule.
Problem 3.7: For a van der Waals gas, plot the isotherms in the in the

P

V plane (

P and

V
are the reduced pressure and volume) for reduced temperatures

T = 0.5,

T = 1.0, and

T = 1.5.
For

T = 0.5, is

P = 0.1 the equilibrium pressure of the liquid gas coexistence region?
Solution 3.7: We use Maple to plot the curves. The graphs have the form We see that for

T = 0.5 there is no stavle region at all. To illustrate the situation we plot the curves in the
vicinity of

T = 1.
Problem 3.8: Consider a binary mixture composed of two types of particles, A and B. For this
system the fundamental equation fog the Gibbs energy is
G = n
A

A
+n
B

B
, (3.1)
the combined rst and second laws are
dG =SdT +V dP+
A
dn
A
+
B
dn
B
(3.2)
(S is the total entropy and V is the total volume of the system), and the chqemical potentials
A
and
B
are intensive so that

A
=
A
(P, T, x
A
) and
B
(P, T, x
B
).
38 CHAPTER 3. THE THERMODYNAMICS OF PHASE TRANSITIONS
4
2
0
2
4
6
8
10
12
0.6 0.7 0.8 0.9 1 1.1 1.2
v
Figure 3.2: The upper curve corresponds

T = 1.5, the lower one - to

T = 0.5.
0
0.2
0.4
0.6
0.8
1
1.2
1.4
1.6
1.8
2
P
1 2 3 4 5
v
Figure 3.3: The upper curve corresponds

T = 1.05, the lower one - to

T = 0.85.
Use these facts to derive the relations
sdT vdP+

=A,B
x

= 0 (3.3)
and

=A,B
x

(d

+s

dT v

dP) = 0, (3.4)
where s =S/n, n =n
A
+n
B
, s

= (S/n

)
P,T,n
,=
, and v

= (V/n

)
P,T,n
,=
with =A, B and
= A, B.
Solution 3.8: Let us express n

as n

= nx

and divide Eq. (3.1) by n. We get


dg = d
_

_
=

(x

dx

) (3.5)
39
The let us divide Eq. (3.2) by n to obtain
dg =sdT +vdP+

dx

. (3.6)
Equating the right hand sides of these equation we prove Eq. (3.3). Further, the entropy is an
extensive variable. Consequently,is should be a homogeneous function of n

.
S =

(S/n

)
P,T,n
,=
n

= n

.
Substituting these expressions into Eq. (3.3) we prove Eq. (3.4).
Problem 3.9: Consider liquid mixture (l) of particles A and B coexisting in equilibrium with
vapor mixture (g) of particles A and B. Show that the generalization of of the Clausius-Clapeyron
equation for the coexistence curve between the liquid and vapor phases when the mole fraction
of of A in the liquid phase is xed is given by
_
P
T
_
x
l
A
=
x
g
A
(s
g
A
s
l
A
) +x
g
B
(s
g
B
s
l
B
)
x
g
A
(v
g
A
v
l
A
) +x
g
B
(v
g
B
v
l
B
)
, (3.7)
where s

= (S/n

)
P,T,n
,=
, and v

= (V/n

)
P,T,n
,=
with = A, B and = A, B. [Hint:
Equation (b) of the Problem (3.8) is useful.]
Solution 3.9: Let us apply Eq. (3.4) to the gas phase. We get

x
g

_
d
g

+s
g

dT v
g

dP
g
_
= 0.
Dividing this equation by dT we obtain

x
g

_
d
g

dT
+s
g

v
g

dP
g
dT
_
= 0.
Now, let us take into account that at the coexistence curve
g

=
l

. Thus
d
g

dT
=
d
l

dT
=s
l

+v
l

dP
l
dT
.
Since at the equilibrium P
g
= P
l
, we prove Eq. (3.7).
Problem 3.10: A PVT system has a line of continuous phase transitions (a lambda line) sepa-
rating two phases, I and II, of the system. The molar heat capacity c
P
and the thermal expansivity

P
are different in the two phases. Compute the slope (dP/dT)
coex
of the line in terms of the
temperature T, the molar volume v, c
P
= c
I
P
c
II
P
, and
P
=
I
P

II
P
.
40 CHAPTER 3. THE THERMODYNAMICS OF PHASE TRANSITIONS
Solution 3.10: At a continuous phase transition the entropy is continuous, s = 0. At the same
time, s = s(P, T) and along the coexistence curve P is a function of the temperature. For each
phase, we get
d s
dT
=
s
T
+
s
P
dP
dT
.
We know that (S/T)
P
= c
P
/T and from the Maxwell relation
(s/P)
T
=(v/T)
P
.
Since s = 0,
(c
P
)/T = (dP/dT)(v/T)
P
= (dP/dT)v(
P
).
The answer is
_
dP
dT
_
coex
=
c
P
T
P
.
Problem 3.11: Water has a latent heat of vaporization, h = 540 cal/gr. One mole of steam is
kept at its condensation point under pressure at T
1
= 373 K. The temperature is then lowered to
T
2
= 336 K. What fraction of the steam condenses into water? (Treat the steam as an ideal gas
and neglect the volume of the water.)
Solution 3.11: Let us use the Clausius-Clapeyron equation for the case of liquid-vapor mixture.
Since v
g
v
l
we get
_
dP
dT
_
coex
=
q
Tv
g
.
Here q is the latent heat per mole. Considering vapor as an ideal gas we get
dP
dT
=
qP
RT
2
P = P
0
e
q/RT
.
As a consequence,
P
1
P
2
= exp
_
q(T
1
T
2
)
RT
1
T
2
_
.
Since the volume is kept constant,
n
g
1
n
g
2
=
P
1
P
2
= exp
_
q(T
1
T
2
)
RT
1
T
2
_
= exp
_
18h(T
1
T
2
)
RT
1
T
2
_
.
Here we have taken into account that the olecular weight of H
2
O is 18. The relative number of
condensed gas is then
n
1
n
2
n
1
= 1exp
_

18h(T
1
T
2
)
RT
1
T
2
_
.
41
Problem 3.12: A liquid crystal is composed of molecules which are elongated (and often have
at segments). I behaves like a liquid because the locations of the center of mass of the molecules
have no long-range order. It behaves like crystal because the orientation of the molecules does
have long range order. The order parameter of the liquid crystal is given by the diatic
S = [nn(1/3)I] ,
where n is a unit vector (called the director), which gives the average direction of alignment of
the molecules. The free energy of the liquid crystal can be written as
=
0
+
1
2
AS
i j
S
i j

1
3
BS
i j
S
jk
S
ki
+
1
4
CS
i j
S
jk
S
kl
S
kl
, (3.8)
where A = A
0
(T T

), A
0
, B and C are constants, I is the unit tensor so
x
i
I x
j
=
i j
, S
i j
= x
i
S x
j
,
and the summation is over repeated indices. The quantities x
i
are the unit vectors
x
1
= x, x
2
= y, x
3
= z.
(a) Perform the summations in the expression for and write in terms of , A, B, C.
(b) Compute the critical temperature T
c
, at which the transition from isotropic liquid to liquid
crystal takes place.
(c) Compute the difference between the entropies between the isotropic liquid ( = 0) and the
liquid crystal at the critical temperature.
Solution 3.12:
(a) First, for brevity, let us express the matrix S as ( s I/3), where
s = nn =
_
_
n
2
1
n
1
n
2
n
1
n
3
n
2
n
1
n
2
2
n
2
n
3
n
3
n
1
n
3
n
2
n
2
3
_
_
.
The Eq. (3.8) can be expressed as
=
0
+
1
2
Aa
2

1
3
Ba
3

3
+
1
4
Ca
4

4
,
a
2
= Tr
_
s
1
3
I
_
2
, a
3
= Tr
_
s
1
3
I
_
3
, a
4
=
_
Tr
_
s
1
3
I
_
2
_
2
. (3.9)
It is easy to calculate b
m
= Tr
_
s
1
3
I
_
m
. Indeed,
Tr
_
s
1
3
I
_
m
=
m

k=0
m!
k!(mk)!
(1)
k
3
k
Tr
_
s
k
I
mk
_
.
42 CHAPTER 3. THE THERMODYNAMICS OF PHASE TRANSITIONS
It is straightforward that for m > 1 and k ,= 0 we get
Tr
_
s
k
I
mk
_
= Tr
_
s
k
I
_
= Tr
_
s
k
_
= 1, Tr I
m
= Tr I = 3.
Here we have used the properties of trace and of s operators. Since trace is independent of
the presentation, let us direct the axis 1 along the vector n. Then
s =
_
_
1 0 0
0 0 0
0 0 0
_
_
Tr
_
s
k
_
= Tr s = 1.
Consequently,
b
m
=
_
1
1
3
_
m
+2
_
1
3
_
m
, a
2
= b
2
=
2
3
, a
3
= b
3
=
2
9
, a
4
= b
2
2
=
4
9
.
Finally, we get
=
0
+
1
3
A
2

2
27
B
3
+
1
9
C
4
. (3.10)
(b) At the transition point the dependence () has two equal minima. To nd this point, let
us shift the origin of by some
0
in order to kill odd in
0
items. It we put =
0
+
we have

0
=
1
3
A(+
0
)
2

2
27
B(+
0
)
3
+
1
9
C(+
0
)
4
. (3.11)
In order to kill the term proportional to
3
one has to put (see the Maple le)
0
= B/6C.
Substituting this value into the expression for the coefcient ,
(2/3)A
0
(2/9)B
2
0
+(4/9)C
3
0
,
and equating the result to 0 we get A = B
2
/27C. Since A = A
0
(T T

) we have
T
c
= T

+
B
2
27A
0
C
.
(c) Since S =/T =(1/3)A
0

2
we get
S =(1/3)A
0
(
+

)(
+
+

) =(2/3)A
0

0
(
+

),
where

are the roots of the equation / = 0 while

are the roots of the equation


/ = 0. Note that the potential () is symmetric, thus
+
+

= 0. The important
part () at the critical point can be obtained substituting
0
and A into Eq. (3.11). It has
the form,
() =
C
9
_

B
2
18C
2

2
_
.
43
Thus,

=
B
6C
,
+

=
B
3C
.
As a result,
S =
1
27
A
0
B
2
C
2
.
Problem 3.13: The equation of state of a gas is given by the Berthelot equation
_
P+
a
Tv
2
_
(v b) = RT .
(a) Find values of the critical temperature T
c
, the critical molar volume v
c
, and the critical
pressure, P
c
in terms of a, b, and R.
(b) Does the Berthelot equation satisfy the law of corresponding states.?
Find the critical exponents , , and from the Berthelot equation.
Solution 3.13:
(a) Let us express the pressure as a function of volume,
P =
RT
v b

a
Tv
2
.
For the critical temperature we have
_
P
V
_
T
=
RT
c
(v
c
b)
2
+
2a
T
c
v
3
c
= 0,
_

2
P
V
2
_
T
=
2RT
c
(v
c
b)
3

6a
T
c
v
4
c
= 0.
From this we obtain:
v
c
= 3b, T
c
=
_
8a
27Rb
, P
c
=
RT
c
8b
.
(b) We put
P = P
c


P, T = T
c


T , v = v
c
v
to get:
_

P+
3

T v
2
_
(3 v 1) = 8

T .
44 CHAPTER 3. THE THERMODYNAMICS OF PHASE TRANSITIONS
(c) First let us simplify the equation of state near the critical point. Putting

P = 1+ p, v = 1+,

T = 1+
and expanding in p, , and up to lowest non-trivial order (see Maple le), we get
p =
3
2

3
12 +7. (3.12)
The derivative (p/)

is
_
p

=
9
2

2
12. (3.13)
Since this function is even in the Maxwell construction

l
dp =

_
p

d = 0
requires
l
=
g
. Since along the coexistence curve p(
g
, ) = p(
g
, ) we get

g
=

8 = 1/2.
Substituting this value of
g
into Eq. (3.13) we observe that
_
p

= 24 = 1.
To obtain the critical exponent let us introduce = 1/ v, the equation of state being

P(,

T) =
8

T
3

3
2

T
.
Expanding near the critical point

P =

T = = 1 we get

P = 1+(3/2)(1)
3
= 3.
3.1 Mini-tests
3.1.1 A
Properties of Van der Waals (VdW) liquid
(i) Find the critical temperature T
c
at which the Van der Waals isotherm has an inection
point. Determine the pressure P
c
and volume, V
c
, for a system of N particles at T = T
c
.
3.1. MINI-TESTS 45
(ii) Express the VdW equation in units of T
c
, P
c
, and V
c
. Show that it has the form
_
P
/
+
3
V
/2
_
(3V
/
1) = 8T
/
(3.14)
where P
/
P/P
c
, V
/
V/V
c
, and T
/
T/T
c
(iii) Analyze the equation of state (3.14) near the critical point. Assume that
P
/
= 1+ p, T
/
= 1+ V
/
= 1n
and show that for small p, and n the equation of state has the approximate form
p = 4 +6n+(3/2)n
3
. (3.15)
(iv) Plot p(n, ) versus n for =0.05 and discuss the plots.
(v) Using the above equation nd the stability region. Show this region in the plot.
(vi) Show that the Maxwell relation can be expressed as

n
r
n
l
n(p/n)

dn = 0 (3.16)
along the equilibrium liquid-gas line. Using this relation and the equation of state nd n
l
and n
r
.
(vii) Discuss why the stability condition and Maxwell relation lead to different stability criteria?
Illustrate discussion using the plot.
Solution
(i) Writing the VdW equation as
P =
NkT
V Nb

N
2
a
V
2
and requiring
_
P
V
_
T
c
= 0,
_

2
P
V
2
_
T
c
= 0
we obtain
T
c
=
8
27
a
bk
, V
c
= 3Nb, P
c
=
1
27
a
b
2
.
(ii) The result is straightforward in one substitutes P = P
c
P
/
, V =V
c
V
/
and T = T
c
T
/
.
(iii) Straightforward.
(iv) Straightforward.
46 CHAPTER 3. THE THERMODYNAMICS OF PHASE TRANSITIONS
(v) The stability region is determined by the equation (p/n)

= 0. We have
(p/n)

= 6 +(9/2)n
2
.
Consequently
n
st
=2
_
/3.
(vi) Since in the main approximation (p/n)

is an odd function of n, n
l
= n
r
. Using
Eq. (3.15) we get (see the Maple le)
n
r
=

4.
3.1.2 B
Curie-Weiss theory of a magnet
The simplest equation for a non-ideal magnet has the form
m = tanh[(Jm+h)] (3.17)
where = 1/kT, J is the effective interaction constant, while h is the magnetic eld measured in
proper units.
(i) Consider the case h = 0 and analyze graphically possible solutions of this equation.
Hint: rewrite equation in terms of an auxiliary dimensional variable m Jm. Show that
a spontaneous magnetization appears at T < T
c
= J/k.
(ii) Simplify Eq. (3.17) at h = 0 near the critical temperature and analyze the spontaneous
magnetization as a function of temperature.
Hint: Put T = T
c
(1+) and consider solutions for small m and .
(iii) Analyze the magnetization curve m(h) near T
c
. Hint: express Eq. (3.17) in terms of the
variables m and

h h h/J and plot

h as a function of m.
(iv) Plot and analyze the magnetization curves for T/T
c
= 0.6 and T/T
c
= 1.6.
Solution
(i) At h = 0 the equation (3.17) has the form:
(J)
1
m = tanh m.
From the graph of the function F( m) (J)
1
mtanh m (see the Maple le) one observes
that only trivial solution m=0 exists at J <11, or at T T
c
=J/k. At T <T
c
there exists
2 non-trivial solutions corresponding spontaneous magnetization. Thus (J)
1
= T/T
c
3.1. MINI-TESTS 47
(ii) The dimensionless function F( m) (T/T
c
) mtanh m, near T
c
acquires the form F( m) =
m+ m
3
/3. Thus the solutions have the from
m = 0,

3.
Since near T
c
the ratio T
c
/T can be substituted by 1, the same result is true for m.
(iii) In general, Eq. (3.17) can be rewritten as
(T/T
c
) m = tanh( m+

h) m+

h = artanh[(1+) m] .
Consequently,

h = artanh[(1+) m] m
To plot the magnetization curve one has to come back to the initial variables.
48 CHAPTER 3. THE THERMODYNAMICS OF PHASE TRANSITIONS
Chapter 4
Elementary Probability Theory and Limit
Theorems
Quick access: 1 2 3 4 5 6 7 8 9 10 11 12 13 14 15
Problem 4.1: A bus has 9 seats facing forward and 8 seats facing backward. In how many
ways can 7 passengers be seated if 2 refuse to ride facing forward and 3 refuse to ride facing
backward?
Solution 4.1: Three people refuse ride facing backward, and they should be denitely placed
to 9 seats facing forward. The number of ways to do that is 9 8 7. In a similar way, 2 people can
occupy 8 seats facing backward in 8 7 ways. Now 175 = 12 seats left. They can be occupied
by 732 = 2 nice people in 12 11 ways. As a result, we get
9 8 78 712 11 = 3725568
ways.
Problem 4.2: Find the number of ways in which 8 persons can be assigned to 2 rooms (A and
B) if each room must have at least 3 persons in it.
Solution 4.2: The number of persons in the room is between 3 and 5. Let us start with the
situation where room A has 3 persons. The number of ways to do that is 8 7 6. The total
number of ways is then
8 7 6+8 7 6 5+8 7 6 5 4 = 8736.
49
50 CHAPTER 4. ELEMENTARY PROBABILITY THEORY . . .
Problem 4.3: Find the number of permutations of the letters in the word MONOTONOUS. In
how many ways are 4 Os together? In how many ways are (only) 3 Os together?
Solution 4.3: The total number of the letters is 10, we have 4 Os, 2 Ns, and other letters do not
repeat. Thus the total number of permutations is 10!/(4!2!) = 75600. To get the number of ways
to have 4 Os together we have to use model word MONTNUS which has 7 letters with 2 repeated
Ns. Thus the number of ways is 7!/2! = 2520. To have three Os together we have 6 places to
insert a separate O to the word MONTNUS, thus the number of ways is 62520 = 15120.
Problem 4.4: In how many ways can 5 red balls, 4 blue balls, and 4 white balls be placed in a
row so that the balls at the ends of the row are of the same color?
Solution 4.4: Let us start with 5 red balls and choose 2 of them to be at the ends. The rest
5+4+42 =11 balls are in the middle. They have to be permuted, but permutation of the balls
of the color must be excluded. Among them the have 3 rest red balls, 4 blue balls and 4 white
balls. Thus we have 11!/(3!4!4!) = 11550 ways. Now let us do the same with white balls. Now
we have 2 rest white balls, but 5 blue balls to permute. We get 11!/(2!5!4!) = 6930 ways. The
same is true for blue balls. The total number of ways is 138600+283160 = 25410 ways.
Problem 4.5: Three coins are tossed.
(a) Find the probability of getting no heads.
(b) Find the probability of getting at least one head.
(c) Show that the event heads on the rst coin and the events tails on the last coin are
independent.
(d) Show that the event only two coins heads and the event three coin heads are dependent
and mutually exclusive.
Solution 4.5:
(a) The answer is (1/2)
3
= 1/8.
(b) The sum of the probabilities is 1. Since the probability to get no heads is 1/8, the answer
is 11/8 = 7/8.
(c) We assume that result of coin tossing is fully random.
(d) The event 3 heads contradicts the requirement only 2.
51
Problem 4.6: Various 6 digit numbers can be formed by permuting the digits 666655. All
arrangements are equivalently likely. Given that the number is even, what is the probability that
two ves are together?
Hint: You must nd a conditional probability.
Solution 4.6: Since the number must be even, the last number is 6, and there are 4 ways to
choose this number. There are 5 digits left, and the total number of their permutations give 5!.
We have to divide it by 3! of permutations of 6 and 2! permutations between 5. We end at the
number of different numbers is 4 5!/3!2! = 40. If we want to have two ves together, it is the
same as permute 4 units, the number of permutations 4 4!/3! = 16. Thus the probability is
P = 16/40 = 2/5.
Problem 4.7: Fifteen boys go hiking. 5 get lost, 8 get sunburned, and 6 return home without
problems.
(a) What is the probability that a sunburned boy got lost?
(b) What is the probability that a lost boy got sunburned?
Solution 4.7: See. pp. 176-177 of the book [1].
Let us dene the event to get sunburned as A, and the event to get lost - B. The probability to
get sunburned is P(A) = 8/15, while the probability to get lost is P(B) = 5/15 = 1/3. Thus
We know that 15 6 = 9 boys had problems. Thus, the joint probability to be either sun-
burned or lost is, P(AB) = 9/15. Thus the probability to be both sunburned and lost is
P(AB) = P(A) +P(B) P(AB) = (8+59)/15 = 4/15.
Consequently, conditional probabilities are
P(A[B) = P(AB)/P(A) = 1/2 (a), P(B[A) = P(AB)/P(B) = 4/5 (b) ,
respectively.
Problem 4.8: A stochastic variable X can have values x = 1 and x = 3. A stochastic variable
Y can have values y = 2 and y = 4. Denote the joint probability density
P
X,Y
(x, y) =

i=1,3

j=2,4
p
i, j
(x i)(y j).
Compute the covariance of X and Y for the following 2 cases:
(a) p
1,2
= p
1,4
= p
3,2
= p
3,4
= 1/4;
(b) p
1,2
= p
3,4
= 0 and p
1,4
= p
3,2
= 1/2.
For each case decide if X and Y are independent.
52 CHAPTER 4. ELEMENTARY PROBABILITY THEORY . . .
Solution 4.8:
(a) First let us calculate
XY) = (1 2+1 4+3 2+3 4)/4 = 6.
At the same time X) = 2 and Y) = 3. Since XY) =X) Y) the variables are indepen-
dent.
(b) We have,
XY) = (1 4+3 2)/2 = 5.
It is not equal to X) Y), so the variables are dependent.
Problem 4.9: The stochastic variables X and Y are independent and Gaussian distributed with
rst moment x) =y) = 0 and standard deviation
X
=
Y
= 1. Find the characteristic function
for the random variable Z = X
2
+Y
2
, and compute the moments z), z
2
) and z
3
). Find the rst
3 cumulants.
Solution 4.9: First let us nd the distribution function for Z. Let us do the problem in two
ways.
Simple way:
Let us transform the problem to the cylindrical coordinates, x, y , . We have
P(x)P(y) =
1
2
e
(x
2
+y
2
)
=
1
2
e

2
, dxdy = dd, .
Thus,
P
Z
(z) =
1
2

dde

2
/2
(z
2
) =
1
2
e
z/2
.
General way:
P
Z
(z) =

dxP
X
(x)

dyP
Y
(y)(x
2
+y
2
z)
=

z
dx
2

z x
2
P
X
(x)

P
Y
_

_
z x
2
_
=
1


z
0
dx

z x
2
exp
_

x
2
2

z x
2
2
_
=
1
2
e
z/2
.
Now, the characteristic function is
f
Z
(k) =e
ikz
) =
1
2


0
dze
(ik1/2)z
=
1
12ik
.
53
Then,
z
n
) = (i)
n
d
n
f
Z
(k)
dk
n

k0
.
Performing calculations (see the Maple le), we obtain:
z) = 2, z
2
) = 8, z
3
) = 48.
Thus,
C
1
(Z) = z) = 2, , C
2
(Z) =z
2
) z)
2
= 4,
C
3
(Z) = z
3
) 3z)z
2
) +2z)
3
= 483 2 8+3 2
3
= 24.
Problem 4.10: A die is loaded so that even numbers occur 3 times as often as odd numbers.
(a) If the die is thrown N = 12 times, what is the probability that odd numbers occur 3 times?
If is is thrown N = 120 times, what is the probability that odd numbers occur 30 times?
Use the binomial distribution.
(b) Compute the same quantities as in part (a), but use the Gaussian distribution
Note: For parts (a) and (b) compute your answers to four places.
(c) Compare answers (a) and (b). Plot the binomial and Gaussian distributions for the case
N = 12.
Solution 4.10:
(a) For the random process, the parial probability for an odd number p = 1/4 and for an even
number is q = 1p = 3/4. The probability to have k odd numbers, according to binimial
distribution is
P
b
N
(k) =
N!
k!(Nk)!
p
k
q
Nk
.
Consequently (see Maple le),
P
b
12
(3) =
12!
3!9!
3
9
4
12
= 0.2881.
In a similar way, P
b
120
(30) = 0.08385.
(b) The number of odd numbers is k) = N/4 and the dispersion is = Np q = 3N/16. Thus
P
G
N
(k) =
1
_
3N/8
exp
_

(k N/4)
2
3N/8
_
.
As a result,
P
G
12
(3) = 0.2660, P
G
120
(30) = 0.0841.
(c) Plots are shown in the Maple le.
54 CHAPTER 4. ELEMENTARY PROBABILITY THEORY . . .
Problem 4.11: A book with 700 misprints contains 1400 pages.
(a) What is the probability that one page contains no mistakes?
(b) What is the probability that one pages contains 2 mistakes?
Solution 4.11: n mistakes can be distributed between g pages by
w
g
(n) =
(g+n1)!
(g1)!n!
.
If one has a page without mistakes, it means that n mistakes are distributed between g1 pages.
The probabiliy of that event is
P(0) =
(g+n2)!
(g2)!n!
n!(g1)!
(g+n1)!
=
g1
g+n1
.
In our case g = 1400 and n = 700, thus P(0) = 1399/2099 0.67. In a similar way, if we have
two misprints at a page, then n 2 misprints are distributed between g 1 pages. Thus, the
number of ways to do that is w
g1
(n2). Consequently,
P(2) =
w
g1
(n2)
w
g
(n)
=
(g+n4)!
(g2)!(n2)!
n!(g1)!
(g+n1)!
=
(g1)n(n1)
(g+n1)(g+n2)(g+n3)
.
We get P(2) 0.07.
Problem 4.12: There old batteries and a resistor, R, are used to construct a circuit. Each battery
has a probability p to generate voltage V = v
0
and has a probability 1 p to generate voltage
V = 0 Neglect internal resistance in the batteries.
Find the average power, V
2
)/R, dissipated in the resistor if
(a) the batteries are connected in the series and
(b) the batteries are connected in parallel.
In cases (a) and (b), what would be the average power dissipated in all batteries were
certain to generate voltage v
0
.
(c) How you realize the conditions and results in this problem in the laboratory?
55
Solution 4.12: Let us start with series connection and nd the probability to nd a given value
of the voltage. We have 4 possible values of the voltage. In units v
0
they are 0, 1, 2, and 3. We
have,
P(0) = (1p)
3
, P(1) = 3p(1p)
2
, P(2) = 3p
2
(1p), P(3) = p
3
.
Consequently,
W) =V
2
)/R =(v
2
0
/R)
_
3p(1p)
2
+12p
2
(1p) +9p
3

= (3v
2
0
/R) p(1+2p).
Now let us turn to parallel connection. Let us for a while introduce conductance G
i
= 1/R
i
of
the batteries, as well as conductance G = 1/R of the resistor. The voltage at the resistor is given
by the expression
V =

i=1,2,3
E
i
G
i
G+
i=1,2,3
G
i
.
Here E
i
are electro-motive forces of the batteries. Now the crucial point is to assume something
about the resistances of the batteries. Let us for simplicity assume that they are equal, and
G
i
G. Actually, it is a very bad assumption for old batteries. Under this assumption, V =
(E
1
+E
2
+E
3
)/3. The the result is just 9 times less that that for series connection.
To check this kind of phenomena in the lab one has to nd many old batteries at a junk-yard,
rst check distribution of their voltages and then make circuits.
The problem is badly formulated.
Problem 4.13: Consider a random walk in one dimension. In a single step the probability of
displacement between x and x +dx is given by
P(x)dx =
1

2
2
exp
_

(x a)
2
2
2
_
dx. (4.1)
After N steps the displacement of the walker is S = X
1
+X
3
+X
N
where X
i
is the displacement
after ith step. After N steps,
(a) what is the probability density for the displacement, S, of the walker, and
(b) what is his standard deviation?
Solution 4.13:
(a) By denition,
P
S
(s) =

dx
1
P(x
1
)

dx
2
P(x
2
). . .

dx
N
P(x
N
)(s x
1
x
2
. . . x
N
) .
To calculate this integral we expand -function into Fourier integral
(x) =

dk
2
e
ikx
.
56 CHAPTER 4. ELEMENTARY PROBABILITY THEORY . . .
In this way we get
P
S
(s) =

dk
2
e
iks

dx
1
e
ikx
1
P(x
1
)

dx
1
e
ikx
2
P(x
2
). . .

dx
N
e
ikx
N
P(x
N
)
=

dk
2
e
iks
_

dxe
ikx
P(x)
_
N
=

dk
2
e
iks
[P(k)]
N
. (4.2)
Here
P(k) =

dxe
ikx
P(x)
is the Fourier component of the single-step probability. From Eq. (4.1) we obtain
P(k) = exp
_
ikak
2

2
/2
_
[P(k)]
N
= exp
_
iNkak
2
N
2
/2
_
.
Now,
P
S
(s) =

dk
2
exp
_
ik(s Na) k
2
N
2
/2

=
1

2N
2
exp
_

(s Na)
2
2N
2
_
.
(b) We have (see Maple le),
S) =

dssP
S
(s) = Na, S
2
) S)
2
= N
2
.
Problem 4.14: Consider a random walk in one dimension for which the walker at each step
is equally likely to take a step with displacement anywhere in the interval d a x d +a,
where a < d. Each step is independent of others. After N steps the displacement of the walker is
S = X
1
+X
3
+ +X
N
where X
i
is the displacement after ith step. After N steps,
(a) what is the average displacement, S), of the walker, and
(b) what is his standard deviation?
Solution 4.14: Starting from the denition, we have
S) =

sP
S
(s)ds =

sds

dx
1
P(x
1
)

dx
N
P(x
N
)(s x
1
. . . x
N
)
=

dx
1
P(x
1
)

dx
N
P(x
N
)(x
1
+x
2
+. . . x
N
) = NX) = Nd .
In a similar way,
S
2
) =

dx
1
P(x
1
)

dx
N
P(x
N
)(x
1
+x
2
+. . . x
N
)
2
.
57
We know that
_

i
x
i
_
2
=

i
x
2
i
+

i,=j
x
i
x
j
.
The number of pairs with non-equal i and j is N(N1). After averaging the obtain
S
2
) = NX
2
) +N(N1)X)
2
.
To calculate X
2
) let us specify the normalized probability as
P(x) =
1
2a
_
1, d a x d +a
0, otherwise
.
Then,
X
2
) =
1
2a

d+a
da
x
2
dx = d
2
+a
2
/3.
Summarizing we get (see Maple le)
S
2
) S)
2
= N(d
2
+a
2
/3) +N(N1)d
2
(Nd)
2
= Na
2
/3.
Problem 4.15: Consider a random walk in one dimension. In a single step the probability of
displacement between x and x +dx is given by
P(x)dx =
1

a
x
2
+a
2
dx. (4.3)
Find the probability density for the displacement of the walker after N steps. Does it satisfy the
Central Limit Theorem? Should it?
Solution 4.15: Using the method of the Problem 4.13 we get
P(k) = e
a[k[
P
S
(s) = 2


0
dk
2
e
Nka
cosks =
1

Na
2
Na
2
+s
2
.
It does not satisfy the Central Limit Theorem. It should not because X
2
) is divergent.
58 CHAPTER 4. ELEMENTARY PROBABILITY THEORY . . .
Chapter 5
Stochastic Dynamics and Brownian Motion
Quick access: 1 2 3 4 5 6 7 8 9 10 11 Test 1
Problem 5.1: Urn A has initially 1 red and 1 white marble, and urn B initially has 1 white
and 3 red marbles. The marbles are repeatedly interchanged. In each step of the process one
marble is selected from each urn at random and the two marbles selected are interchanged. Let
the stochastic variable Y denote conguration of the units. There congurations are possible.
They are shown in Fig. 5.1
3
1
2
Figure 5.1: To the problem 5.1
We denote these 3 realizations as y(n) where n = 1, 2 and 3.
(a) Compute the transition matrix Q and the conditional probability matrix P(s
0
[s).
(b) Compute the probability vector, P(s)[, at time s, given the initial condition stated above.
What is the probability that there are 2 red marbles in urn A after 2 steps? After many
steps?
(c) Assume that the realization, y(n), equals to n
2
. Compute the moment, y(s)), and the
autocorrelation function, y(0)y(s)), for the same initial conditions as in part (b).
59
60 CHAPTER 5. STOCHASTIC DYNAMICS ...
Solution 5.1: The congurations are shown in Fig. 5.1. By inspection of the gure we get,
Q
11
= 0, Q
12
= 1, Q
13
= 0,
Q
21
=
1
2

1
4
=
1
8
, Q
22
=
1
2

1
4
+
1
2

3
4
=
1
2
, Q
23
=
1
2

3
4
=
3
8
,
Q
31
= 0, Q
32
=
1
2
, Q
33
=
1
2
.
Thus the transition matrix Q is given by the expression
Q =
_
_
0 1 0
1/8 1/2 3/8
0 1/2 1/2
_
_
,
while the initial condition corresponds to the probability vector
p(0)[ = (0, 1, 0).
Below we proceed as in exercise 5.1 of the book [1].
The following procedure is clear form the Maple printout. First we nd eigenvalues of Q
which are equal to 1, 1/4. The corresponding right eigenvectors are:
[1, 1, [1, 1, 1]], [
1
4
, 1,
_
6,
3
2
, 1
_
], [
1
4
, 1, [4, 1, 2]]
Thus we denote
1
= 1,
2
=1/4,
3
= 1/4, and
[1) = (1, 1, 1), [2) = (6, 3/2, 1), [3) = (1, 2, 3).
The left eigenvectors are just the right eigenvalues of the transposed matrix, Q
T
. We have for
them the Maple result
[
1
4
, 1, [1, 2, 3]], [
1
4
, 1, [1, 2, 1]], [1, 1, [1, 8, 6]]
Thus,
1[ = (1, 8, 6), 2[ = (1, 2, 1), 3[ = (1, 2, 1).
It is easy to prove that
i[k) = n
i

ik
, where n
1
= 15, n
2
= 10, n
3
= 12.
Now we construct matrices P
i
=[i)i[ to get
P
1
=
_
_
1 8 6
1 8 6
1 8 6
_
_
, P
2
=
_
_
6 12 6
3/2 3 3/2
1 2 1
_
_
, P
3
=
_
_
4 8 12
1 2 3
2 4 6
_
_
.
61
Now, P(s
0
[s) = P(s s
0
) with
P(s) =

s
i
n
i
P
i
=
_

_
1
15
+
3
5
(
1
4
)
s
+
1
3
(
1
4
)
s
8
15

6
5
(
1
4
)
s
+
2
3
(
1
4
)
s
2
5
+
3
5
(
1
4
)
s
(
1
4
)
s
1
15

3
20
(
1
4
)
s
+
1
12
(
1
4
)
s
8
15
+
3
10
(
1
4
)
s
+
1
6
(
1
4
)
s
2
5

3
20
(
1
4
)
s

1
4
(
1
4
)
s
1
15
+
1
10
(
1
4
)
s

1
6
(
1
4
)
s
8
15

1
5
(
1
4
)
s

1
3
(
1
4
)
s
2
5
+
1
10
(
1
4
)
s
+
1
2
(
1
4
)
s
_

_
As it should be, P(0) = I. In this way we solve the part (a).
Now we come to the point (b). Straightforward calculation gives
P(s)[ = p(0)[P(s)
=
_
1
15

3
20
(
1
4
)
s
+
1
12
(
1
4
)
s
,
8
15
+
3
10
(
1
4
)
s
+
1
6
(
1
4
)
s
,
2
5

3
20
(
1
4
)
s

1
4
(
1
4
)
s
_
.
We are asked to nd the probability of the conguration 3. After 2 steps it is 3/8, and at s
we get 2/5.
The part (c) is calculated in a straightforward way. We obtain
y(s)) =
29
5

3
10
(
1
4
)
s

3
2
(
1
4
)
s
,
y(0)y(s)) =
79
50
(1)
(1+s)
4
(s)

33
4
16
(s)
+
81
100
(1)
(1+2s)
16
(s)
+
11
2
(1)
(1+s)
16
(s)

3
2
4
(s)
+
841
25
Problem 5.2: Three boys, A,B, and C, stand in a circle and play catch (B stands to the right of
A). Before throwing the ball, each boy ips a coin to decide whether to throw to the boy on his
right or left. If heads comes up, the boy throws to his right. If tails come up, the boy throws
to his right.
The coin of boy A is fair (50% heads and 50% tails), the coin of boy B has head of both
sides, and the coin of boy C is weighted (75% heads and 25% tails).
(a) Compute the transition matrix, its eigenvalues, and its left and right eigenvalues.
(b) If the ball is thrown in regular intervals, approximately what fraction of time each boy have
the ball (assuming they throw the ball many times)?
(c) If boy A has the ball to begin with, what is the chance it has it after 2 throws? What is the
chance he will have in after s throws.
62 CHAPTER 5. STOCHASTIC DYNAMICS ...
Solution 5.2: The problem is similar to the previous one. Let us map the notations
A 1, B 2, C 3,
and note that right means counterclockwise. Then
Q
12
= Q
13
= 1/2, Q
23
= 1, Q
21
= 0, Q
31
= 3/4, Q
32
= 1/4.
At the same time O
11
= Q
22
= Q
33
= 0. As a result, the transition matrix has the form
Q =
_
_
0 1/2 1/2
0 0 1
3/4 1/4 0
_
_
.
The following solution is similar to the previous problem (see the Maple printout). Note that the
eigenvalues and eigenvectors are complex, but conjugated. It means that the probability is real,
but oscillates in time).
The answers are: after many throws the chance for the boy A to hold the ball is 6/19; starting
from the given initial conditions his chance to hold the ball after 2 throws is 3/8.
Problem 5.3: A trained mouse lives in the house shown in Fig. 5.2.
C (3)
B (2)
A (1)
Figure 5.2:
A bell rings at regular intervals (short compared with the mouses lifetime). Each time it rings,
the mouse changes rooms. When he changes rooms, he is equally likely to pass through any of
the doors of the room he is in.
Let the stochastic variable Y denotes mouse in a particular room. There 3 realizations of
Y:
1. Mouse in room A
2. Mouse in room B
3. Mouse in room C
63
which we denote as y(1), y(2), and y(3), respectively.
(a) compute the transition matrix, Q, and the conditional probability matrix, P(s
0
[s).
(b) . Compute the probability vector, P(s)[, at time s, given that the mouse starts in room C.
Approximately, what fraction of his life does he spend in each room?
(c) Assume that the realization, y(n) equals to n. Compute the moment, y(s)), and the auto-
correlation function, y(0)y(s)), for the same initial condition as in part (b).
Solution 5.3: The problem is very much similar to the previous ones. We have
Q
12
= Q
13
= 1/2, Q
21
= 1/3, Q
23
= Q
32
= 2/3, Q
31
= 1/3, Q
ii
= 0.
Thus
Q =
_
_
0 1/2 1/2
1/3 0 2/3
1/3 2/3 0
_
_
.
Now we do the same as in previous problems (see Maple printout). The answers are obvious
from the printout.
Problem 5.4: The doors in the mouses house shown in Fig. 5.2 so thay get periodically larger
and smaller. This causes the mouses transition between the rooms to become time periodic. Let
the stochastic variable Y have the same meaning as in Problem 5.3. The transition matrix is now
given by
Q
11
(s) = Q
22
(s) = Q
33
(s) = 0, Q
12
(s) = cos
2
(s/2), Q
12
(s) = sin
2
(s/2),
Q
21
(s) = 1/4+(1/2)sin
2
(s/2), Q
23
(s) = 1/4+(1/2)cos
2
(s/2)
Q
31
(s) = (1/2)cos
2
(s/2), Q
32
(s) = (1/2) +(1/2)sin
2
(s/2),
(a) If initially the mouse is in room A, what is the probability to nd it in room A after 2s
room changes?
(b) If initially the mouse is in room B, what is the probability to nd it in room A after 2s room
changes? In room B?
Solution 5.4: The solution is rather tedious, but obvious from the Maple le.
64 CHAPTER 5. STOCHASTIC DYNAMICS ...
Problem 5.5: Consider a discrete random walk on a one-dimensional periodic lattice with
2N+1 sites (label the sites from N to N). Assume that the walker is equally likely to move on
the lattice site to the left or right at each step. Treat this problem as a Markov chain.
(a) Compute the transition matrix, Q, and the conditional probability, P(s
0
[s).
(b) Compute the probability P
1
(n, s) at time s, given the walker starts at site n = 0.
(c) If the lattice has 5 sites (N = 2), compute the probability to nd the walker on each site
after s = 2 steps and after s = steps.Assume that the walker starts at site n = 0.
Hint: Let us understand the word periodic as peridically extended, i. e. possessing the property
P[n+(2N+1), t] = P(n, t).
Solution 5.5: The transition probabilities are given by the expressions
w
n1,n
= w
n,n1
= w
n+1,n
= w
n,n+1
= 1/2.
The Master equation has the form
dP
n
/ds = w
n1,n
P
n1
+w
n+1,n
P
n+1
P
n
(w
n,n1
+w
n,n+1
)
= (1/2)(P
n1
+P
n+1
2P
n
) .
Let us look for the solution in the form
P
n
(s) e
s+in
.
Then we get
=[1cos()] =2sin
2
(/2).
Now we have to nd . If we apply cyclic boundary conditions (assuming that the word periodic
in the problem formulation means that) we have to require: P(n+2N+1) = P(n) at any s. Thus
e
i(2N+1)
= 1 (2N+1) = 2k,
where k is the integer number ranging from N to N. Thus, the general solution is
P(n, s) =
N

k=N
P
k
exp
_
in
2k
2N+1
2ssin
2
k
2N+1
_
.
The quantities P
k
should be found from the initial conditions. At s = 0 we can rewrite the l. h. s.
of the requirement P(n, 0) =
n,0
in the form
P(n, 0) =
N

k=N
P
k
e
in2k/(2N+1)
.
65
On the other hand,

n,0
=
1
2N+1
N

k=N
e
in2k/(2N+1)
.
From that we immediately get P
k
= const(k). The proper constant 1/N (s) is the normalization
coefcient found from the condition
N

n=N
P(n, s) = 1.
Finally
P(n, s) =
1
N (s)
N

k=N
exp
_
in
2k
2N+1
2ssin
2
k
2N+1
_
.
Here
N (s) =
N

n=N
exp
_
in
2k
2N+1
2ssin
2
k
2N+1
_
= 2N+1.
Now let us nd the conditional probability P(n, s[m, 0). Now we have to apply the initial condi-
tion P(n, 0) =
m,0
, or
P(n, 0) =
N

k=N
P
k
e
in2k/N
=
n,m
=
N

k=N
P
k
e
i(nm)2k/(2N+1)
.
Thus,
P
k
= e
im2k/(2N+1)
.
Consequently,
P(s s
0
[0)
mn
= P(n, s[m, 0) =
1
2N+1
N

k=N
exp
_
i(nm)
2k
2N+1
2ssin
2
k
2N+1
_
.
As s the probability tends to 1/(2N +1). The calculation for N = 2 is straightforward, see
Maple le. Finally, the matrix Q
mn
= P(1[0)
mn
.
Problem 5.6: At time t, a radioactive sample contains n identical undecayed nuclei, each with
the probability for the unit time, , of decaying. The probability of decay during the time t
t +t is t. Assume that at time t = 0 there are n
0
undecayed nuclei present.
(a) Write down and solve the master equation of the process [nd P
1
(n, t)].
(b) Compute the the mean number of undecayed nuclei and the variance as a function of time.
(c) What is the half-life of this decay process?
66 CHAPTER 5. STOCHASTIC DYNAMICS ...
Solution 5.6: The master equation has the form
dP
dt
=nP(n, t) +(n+1)P(n+1, t).
This is a linear master equation, and we solve it by the method of generating functional, p. 262
of the book [1]. We introduce
F(z, t) =

n=
z
n
P
1
(n, t).
Then we multiply the master equation by z
n
and sum over n. We get
F
t
=(z 1)
F
z
. (5.1)
Indeed,

n
z
n
(n+1)P(n+1, t) =
d
dz

n
z
n+1
P(n+1, t) =
F
z
.
In a similar way

n
z
n
nP(n, t) = z

n
z
n1
nP(n, t) = z
F
z
Introducing = t we rewrite Eq. 5.1 in the form
F

+(z 1)
F
z
= 0. (5.2)
The characteristics of this equation meet the equation
d =
dz
z 1
ln(z 1) = const .
Consequently,
F(z, t) =
_
e
+ln(z1)
_
=
_
(z 1)e

.
Since P(n, 0) =
n,n
0
we get
(z 1) = z
n
0
(u) = (u+1)
n
0
.
As result,
F(z, t) =
_
(z 1)e

+1

n
0
.
Now,
n(t)) =
_
F
z
_
z=1
= n
0
e

.
In a similar way,
n
2
(t)) n(t))
2
= n
0
e

_
1e

_
.
67
To nd the probabilities let us express the quantity
_
(z 1)e

+1

n
0
= e
n
0

[z +(e

1)]
n
0
= e
n
0

n
0

n=0
n
0
!
n!(n
0
n)!
z
n
(e

1)
n
0
n
.
Thus,
P
1
(n, ) =
n
0
!
n!(n
0
n)!
e
n
0

_
1e

_
n
0
n
.
Problem 5.7: Consider a random walk on the lattice shown in Fig. 5.3. The transition rates are
4
1
2 3
Figure 5.3: A sketch to the problem 5.7.
w
12
= w
13
= 1/2, w
21
= w
23
= w
24
= 1/3, w
31
= w
32
= w
34
= 1/3,
w
42
= w
43
= 1/2, w
14
= w
41
= 0.
(a) Write the transition matrix, W, and show that that this system obey detailed balance.
(b) Compute the symmetric matrix, V, and nd its eigenvalues and eigenvectors.
(c) Write P
1
(n, t) for the case P(n, 0) =
n,1
. What is P(4, t)?
Solution 5.7: The solution is similar to the Exercise 5.2 in p. 245 of the book [1]. The answers
are obvious from the Maple le.
Problem 5.8: Consider a random walk on the lattice shown in Fig. 5.4. The site P absorbs the
walker. The transition rates are
w
12
= w
13
= 1/2 w
21
= w
23
= w
2P
= 1/3,
w
31
= w
32
= w
3P
= 1/3, w
P1
= w
P2
= 1/2.
(a) Write the transition matrix, M, and compute its eigenvalues and eigenvectors.
(b) In the walker starts at site n = 1 at time t = 0, compute the mean rst passage time.
68 CHAPTER 5. STOCHASTIC DYNAMICS ...
P
1
2 3
Figure 5.4: A sketch to the problem 5.8.
Solution 5.8: The solution is similar to the Exercise 5.4 of the book [1]. The answers are clear
from the Maple le.
Problem 5.9: Let us consider on RL electric circuit with resistance, R, and inductance, L,
connected in series. Even there is no average electromotive force (EMF) exists across the resistor,
because of the discrete character of electrons in the circuit and their random motion, a uctuating
EMF, (t), exists whose strength is determined by the temperature, T. This, in turn, induces the
uctuating current, I(t), in the circuit. The Langevin equation for the current is
L
d I(t)
dt
+RI(t) = (t). (5.3)
If the EMF is delta-correlated,
(t
2
)(t
1
)) = g(t
2
t
1
)
and (t))

= 0, compute g and the current correlation function I(t


2
)I(t
1
))

)
T
. Assume that
the average magnetic energy LI
2
0
)
T
/2 = kT/2 and I
0
)
T
= 0 where I(0) = I
0
.
Solution 5.9: Let us divide Eq. (5.3) by R, denote L/R and measure time in units of and
the current in units I
1
R
1

g. Namely, dene = t/, and j(t) = I(t)/I


1
. Then the equation
for the current has the form
d j/d+ j = (), (
2
)(
1
)) = (
1
).
Its solution has the form
j() = j
0
e


0
dse
s
(s).
Multiplying solutions for j(
2
) and for j(
1
) and averaging over realizations of we get
j(
2
) j(
1
))

= j
2
0
e
(
2
+
1
)
+


2
0
ds
2


1
0
ds
1
(s
2
s
1
)e
s
1
+s
2

2
=
_
j
2
0

1
2
_
e

2
+
1
2
e
[
2

1
[
.
69
The rst item must vanish since in the equilibrium the process must be stationary, i. e. it must
depend only on the difference
2

1
. Thus,
j
2
0
)
T
=
1
2
I
2
0
)
T
=
g
2R
2
.
On the other hand,
LI
2
0
) = kT g = 2R
2
kT/L.
Consequently,
j(
2
) j(
1
))

)
T
=
1
2
e
[
2

1
[
, .
In dimensional variables,
I(t
2
)I(t
1
))

)
T
= I
2
1
j(
2
) j(
1
))

)
T
=
kT
L
e
[t
2
t
1
[/
.
Problem 5.10: Due to the random motion and discrete nature of electrons, an LRC series
circuit experience a random electromotive force (EMF), (t). This, in turn, induces a random
varying charge, Q(t), on the capacitor plates and a random current, I)t) = dQ(t)/dt, through the
resistor and inductor. The random charge, Q(t), satises the Langevin equation
L
d
2
Q
dt
2
+R
dQ
dt
+
Q
C
= (t).
Assume that EMF is delta-correlated,
(t
2
)(t
1
)) = g(t
2
t
1
)
and (t))

= 0. Assume that the circuit is at the temperature T and the average magnetic energy
in the inductor and average electric energy in the capacitor satisfy the equipartition theorem,
1
2
LI
2
0
)
T
=
1
2C
Q
2
0
)
T
,
where Q(0) = Q
0
, I(0) = I
0
. Assume that I
0
)
T
= 0, Q
0
)
T
= 0, Q
0
I
0
)
T
= 0.
(a) Compute the current correlation function I(t
2
)I(t
1
))

)
T
.
(b) Compute the variance of the charge distribution, (Q(t) Q
0
)
2
)

)
T
.
70 CHAPTER 5. STOCHASTIC DYNAMICS ...
Solution 5.10: The problem is exactly the same as in the Exercise 5.5 of the book [1]. After
mapping
Q x, I v, L m, LC
2
0
, R
we obtain the same equation. Thus,
I(t +)I(t))

)
T
=
kT
L
e
[[
C([[),
where
C() = cosh (/)sinh, = R/L, =
_

2
0
.
The variance of charge distribution can be calculated by integrating the formal solution
I(t) = I
0
e
t
Q
0

2
0

e
t
sinht +
1
L

t
0
dt
/
(t
/
)e
(tt
/
)
C(t t
/
)
over time:
Q(t) = Q
0
+I
0
1e
t

Q
0

2
0
2
_
1e
()t


1e
(+)t
+
_
+
1
L

t
0
dt
//

t
0
dt
/
(t
/
)e
(t
//
t
/
)
C(t
//
t
/
)
Now we do averages over and T to obtain (linear terms vanish!)
Q
2
(t))

)
T
=Q
2
0
)
T
_
1

2
0
2
_
1e
()t


1e
(+)t
+
__
2
+I
2
0
)
T
_
1e
t

_
2
+
g
L
2

t
0
dt
1

t
0
dt
2

t
0
dt
3

t
0
dt
4
e
(t
1
t
2
+t
3
t
4
)
C(t
1
t
2
)C(t
3
t
4
)(t
2
t
4
)
Now we substitute Q
2
0
)
T
=CkT, I
2
0
)
T
= kT/L, and g = 4RkT, and then apply function. The
last term becomes
4RkT
L
2

t
0
dt
1

t
0
dt
2

t
0
dt
3
e
(t
1
2t
2
+t
3
)
C(t
1
t
2
)C(t
3
t
2
).
The rest integrations must be done explicitly which results in lengthy expressions.
Problem 5.11: Consider a Brownian particle of mass m in one dimension in the presence of
a constant force f
0
in a uid with force constant and in th4e presence of a delta-correlated
random force (t) such that (t
1
)(t
2
))

= g(t
1
t
2
) and (t))

= 0. Assume that the velocity


and displacement of the Brownian particle at t = 0 are v
0
and x
0
, respectively.
(a) Compute the velocity correlation function v(t
1
)v(t
2
))

.
(b) Compute the displacement variance (x(t) x
0
)
2
).
71
Solution 5.11: The differential equation for the particle is
d v(t)
dt
+

m
v(t)
f
0
m
=
1
m
(t),
dx
dt
= v(t).
Its solution is
v(t) = v
0
e
(/m)t
+
f
0

_
1e
(/m)t
_
+
1
m

t
0
dt
/
e
(/m)(t
/
t)
(t
/
).
The quantity
v
d
(t) =
f
0

_
1e
(/m)t
_
is just the drift velocity in the force f
0
. Using results of the Sec. 5.E.1 of the book [1] we obtain
[v(t +) v
d
(t +)] [v(t) v
d
(t)])

)
T
=
kT
m
e
(/m)[[
.
Since v(t))
T
= v
d
(t) we obtain
v(t +)v(t))

)
T
= v
d
(t +)v
d
(t) +
kT
m
e
(/m)[[
.
In a similar way, we cam introduce drift displacement
x
d
(t) =

t
0
dt
/
f
0

_
1e
(/m)t
/
_
=
f
0

_
t
m

_
1e
(/m)t
_
_
.
Finally we obtain
[x(t) x
0
x
d
(t)]
2
)

)
T
=
2kT

_
t
m

_
1e
(/m)t
_
_
.
Tests for training
Problem 5.12: Stochastic processes
Consider an electron (spin S = 1/2) bounded to an impiruty center and assume that the tempera-
ture T is much less than the energy difference between the levels of orbital motion. Consequently,
only spin degrees of freedom are excited.
Let the system be placed in external magnetic eld B, the additional energy in magnetic eld
is B where corresponds to the values 1/2 of the spin component along the eld. Here
=[e[ h/2mc is the Bohr magneton, c is the velocity of light since the Gaussian system is used.
The magnetic moment of the states with S
z
=1/2 is .
(a) Find the stationary probabilities for the states with S
z
=1/2, P

.
- Find average magnetic moment of the system as function of temperature T and magnetic eld
B.
72 CHAPTER 5. STOCHASTIC DYNAMICS ...
(b) Find mean square uctuation of magnetic moment, (M)
2
) and the ratio (M)
2
)/M)
2
.
- Show that
(M
2
)) =

2
cosh
2
(B/T)
. (5.4)
- Discuss limiting cases of large and low temperatures. Explain these results qualitatively.
(c) Write down the Master Equation for the probability P
i
(t) to nd the system in the state i =
at time t.
- Using the result for the stationary case nd the relation between the transition probabilities
W
+
and W
+
for the transition from the state + to the state - and for the reverse transition,
respectively. Express the probabilities W
+
and W
+
through the quantity W =W
+
+W
+
.
- Derive the Master Equation for the population difference, P(t) w
+
(t) w
+
(t).
(d) Write down the Master Equation for the conditional probability, P( f , t[i, t
0
), to nd the system
in the state f at time t under the conditions that at time t
0
it was in the state i.
- Specify initial conditions to this equation.
- Show that solution of this equation with proper initial condition is
P( f , t[i, t
0
) = w
f
+(
i f
w
f
)e
W[tt
0
[
(5.5)
where w
f
is the stationary probability to nd the system in the state f .
Hint: It is convenient to express the set of Master Equations for the quantities P( f , t[i, t
0
) at
different i and f as an equation for the 22 matrix

P with matrix elements P
f i
(t[t
0
) P( f , t[i, t
0
).
The solution of the matrix Master Equation

P/T =

W

P at t t
0
can then be searched in in the
matrix as

P
k

C
(k)
e

k
(tt
0
)
. Here
k
are eigenvalues of the matrix W, while

C
(k)
are time-
independent matrices. They are determined from the intial conditions and from the fact that at
t t
0
the conditional probabilities tend to the stationary ones.
(e) Using Eq. (5.5) show that the correlation function M(t)M(0)) has the form
M(t)M(0)) =(M)
2
)e
W[t[
(5.6)
where (M)
2
) is given by Eq. (5.4) while W =W
+
+W
+
.
- Find the uctuation spectrum.
Solution 5.12: (a) Denote w

the probabilities of the states S


z
=1/2. We have
M =(w
+
w

) . (5.7)
According to the Gibbs distribution,
w
+
/w

= e
2B/T
. (5.8)
73
Since only 2 spin levels are involved,
w
+
+w

= 1. (5.9)
Combinig Eqs. (5.8) and (5.9) we obtain,
w
+
=
1
e
2B/T
+1
, w

=
e
2B/T
e
2B/T
+1
, w

w
+
= tanh
_
B
T
_
(5.10)
As a result,
M = tanh
_
B
T
_
(5.11)
(b) We get:
M
2
) =
2
(w
+
+w

) =
2
M)
2
=
2
tanh
2
(B/T). (5.12)
As a result,
(M
2
)) =

2
cosh
2
(B/T)
,
(M)
2
)
M)
2
=
1
sinh
2
(B/T)
(5.13)
At low temperatures, T B, uctuations are exponentially suppressed because spin is aligned
to magnetic eld.
(c) The equation reads,
w
i
(t)
t
=

s=
[W
is
w
i
(t) W
si
w
s
(t)] (5.14)
- In the stationary situation, using the detailed equilibrium principle,
W
is
w
i
(t) =W
si
w
s
(t)
we obtain
W
+
W
+
=
w

w
+
= e
2B/T
, , W
+
=
W
1+e
2B/T
, W
+
=
W
1+e
2B/T
(5.15)
- Coming back to Eq. (5.14) we get
w
+
(t)
t
= W
+
w
+
(t) +W
+
w

(t),
w

(t)
t
= W
+
w

(t) +W
+
w
+
(t). (5.16)
Subtracting equations we arrive at the result,
P(t)
t
=WP(t) (5.17)
74 CHAPTER 5. STOCHASTIC DYNAMICS ...
(d) We consider a stationary random process, and P( f , t[i, t
0
) is a function of t t
0
. Let us assume
t
0
= 0, t 0 and denote Let us for brevity denote P( f , t[i, 0) P
f i
(t). The Master Equation for
P
f i
(t) has the form
P
f i
(t)
t
=

s
[W
f s
P
f i
(t) W
sf
P
si
(t)] . (5.18)
The initial condition to this equation is
P
f i
(t) =
i f
at t 0. (5.19)
Let us introduce the matrix

P(t) with matrix elements P
i f
(t) = P( f , t[i, 0). Then Eq. (5.18) can
be rewritten as


P(t)
t
=

W

P(t), with

W =
_
W
f s
W
sf
W
f s
W
sf
_
. (5.20)
It is natural to search the solution of Eq, (5.18) in the form

P(t)
k

C
(k)
e

k
t
. Then
k
are the
eigenvalues of the matrix

W while

C
k
are the numerical matrices to be determined from intial
conditions at t = 0 and from the fact that P
f i
(t) w
f
at t . Here w
f
is the stationary
probability of the state f .
Equating the determinant of

W

I we get

1
= 0,
2
=W
f s
+W
sf
W .
Here

I is the unit matrix Then, in general,
P
f i
(t) =C
(1)
f i
+C
(2)
f i
e
Wt
where

C
(k)
are time-independent matrices. From the initial conditions (5.19),
C
(1)
f i
+C
(2)
f i
=
f i
.
On the other hand, at t the conditions probability tends to the stationary one, w
f
. Conse-
quently,
C
(1)
f i
= w
f
.
Finally, because of the time reversibility, P
f i
(t) depends only on [t[. As result,
P
f i
(t) = w
f
+(
f i
w
f
)e
W[t[
This expression coincides with Eq. (5.5).
(e) By denition,
M(t)M(0)) =M(t)M(0)) M)
2
=

i f
M
i
M
f
[P( f , t[i, 0) w
f
]w
i
.
75
Using Eq. (5.5), one can rewrite the above equation as
M(t)M(0)) = e
W[t[

i f
M
i
M
f
(
i f
w
f
)w
i
=(M)
2
)e
W[t[
This expresssion coincides with Eq. (5.6).
The uctuation spectrum is
(M)

dt e
t
M(t)M(0)) =(M)
2
)

dt e
tW[t[
.
The result reads as,
(M)

= 2(M)
2
).
W

2
+W
2
76 CHAPTER 5. STOCHASTIC DYNAMICS ...
Chapter 6
The Foundations of Statistical Mechanics
This chapter will be skipped this year
77
78 CHAPTER 6. THE FOUNDATIONS OF STATISTICAL MECHANICS
Chapter 7
Equilibrium Statistical Mechanics
Quick access: 1 2 3 4 5 6 7 8 9 10 11 12 13 14 15 16 17 18 19 20 21 22 23 24
Problem 7.1: Compute the structure function for N noninteracting harmonic oscillators, each
with frequency and mass m. Assume the system has total energy E. Using this structure
function and the microcanonical ensemble, compute the entropy and the heat capacity of the
system.
Solution 7.1: One can use either classical, or quantum Hamiltonian. Let us do the problem
classically. For brevity let us assume the the oscillators are one-dimensional. Then the Hamilto-
nian is
H =
N

i=1
_
p
2
i
2m
+
m
2
x
2
i
2
_
= E .
The total number of states is
(E) =

V
dx
1

V
dx
2
. . .

V
dx
N

dp
1

dp
2
. . .

dp
N

_
E
N

i=1
_
p
2
i
2m
+
m
2
x
2
i
2
_
_
.
Here
(x) =
_
1, x > 0
0, x < 0
is the Heaviside unit step function. We have
d(x)/dx = (x),
thus the structure function (E) = (E)/E is given by the equation
(E) =

V
dx
1

V
dx
2
. . .

V
dx
N

dp
1

dp
2
. . .

dp
N

_
E
N

i=1
_
p
2
i
2m
+
m
2
x
2
i
2
_
_
.
79
80 CHAPTER 7. EQUILIBRIUM STATISTICAL MECHANICS
Let us use the following trick. Denote q
i
= x
i
_
m
2
/2, i = 1..N, q
i+N
= p
i
_
1/2m and R
2
= E.
Then
=
_
2

_
N

dq
1
. . .

dq
2N

_
R
2

2N

j=1
q
2
j
_

_
2

_
N

(R).
From the dimensionality considerations,

(R) = A
2N
R
2N
.
To nd the coefcient A
2N
let us consider the integral

dq
1
. . .

dq
2N
exp
_

2N

j=1
q
2
j
_
=
_

dqe
q
2
_
2N
=
N
.
On the other hand, the same integral is


0
dR

R
e
R
2
= 2NA
2N


0
dRR
2N1
e
R
2
= NA
2N
(N)
Thus
A
2N
=

N
N(N)
.
Finally,
(E) =
1
N(N)
_
2
E

_
N
, (E) =
1
E (N)
_
2
E

_
N
.
Now, the entropy is
S/k = ln
(E)
N!h
N
= ln
_
E
h
_
N
1
(N!)
2
= Nln(E/ h) 2NlnN+2N.
Here we used the Stirlings formula. Now,
1
T
=
S
E
=
Nk
E
E = NkT, C = kN.
This is the classical answer. The quantum calculation is demonstrated in the Exercise 7.1 of the
book [1].
Problem 7.2: A system consists of N noninteracting, distinguishable two-level atoms. Each
atom can exist in one of two states, E
0
= 0, and E
1
=. The number of atoms in energy level E
1
is n
1
. The internal energy of the system is U = n
0
E
0
+n
1
E
1
.
(a) Compute the entropy of the system as a function of internal energy.
(b) Compute the heat capacity of a xed number of atoms, N.
81
Solution 7.2: Since E
0
= 0 the energy U cam be realized in U/ n
1
particles are in the state
1, and (NU/) = Nn
1
particles are in the state 0. The number of ways to do that is
W =
N!
n
1
!(Nn
1
)!
Consequently,
S
k
= ln
N!
(Nn
1
)!
NlnNNn
1
lnn
1
+n
1
(Nn
1
)ln(Nn
1
) +(Nn
1
).
Finally
S
k
NlnN
U

ln
U


_
N
U

_
ln
_
N
U

_
.
To nd temperature we do usual procedure:
1
T
=
S
U

k

ln
N U
U
.
As result,
U =
N
e
/kT
+1
, C = Nk
_

kT
_
2
e
/kT
_
e
/kT
+1
_
2
.
Problem 7.3: A lattice contains N normal lattice sites and N interstitial lattice sites. N identical
atoms sit on the lattice, M on the interstitial sites and N M on the normal sites (N M 1.
If an atom occupies a normal sites, its energy E = 0. If an atom occupies an interstitial site, its
energy is E = . Compute the internal energy and heat capacity as a function of temperature for
this lattice.
Solution 7.3: The internal energy is U = M, so we have to express M as a function of temper-
ature. The number of ways in which one can place M atoms at N interstitial sites is
W
i
=
N!
M!(NM)!
.
The rest NM atoms can be placed at N normal states in the same number of ways. Thus
W =
_
N!
M!(NM)!
_
2

S
k
= 2ln
N!
M!(NM)!
.
Using the Stilings formula we get
S/k 2[NlnNNMlnM+M(NM)ln(NM) +(NM)]
2[NlnNMlnM(NM)ln(NM)] ,
T
1
= (2k/)ln(NM)/M, M =U/.
Thus,
U =
N
e
/2kT
+1
Ne
/kT
, C 2kN(/2kT)
2
e
/2kT
.
82 CHAPTER 7. EQUILIBRIUM STATISTICAL MECHANICS
Problem 7.4: Consider a lattice with N spin-1 atoms with magnetic moment . Each atom can
be in one of 3 spin states:, S
z
= 1, 0, +1. Let n
1
, n
0
and n
1
denote the respective number of
atoms in each of those spin states. Find the total entropy and the conguration which maximizes
the total entropy. What is the maximum entropy? (Assume that no magnetic eld is present,
so all atoms have the same energy. Also assume that atoms on different lattice sites cannot be
exchanged, so they are distinguishable).
Solution 7.4: We have
W =
N!
n
1
!n
1
!(Nn
1
n
1
)!
,
lnW NlnNn
1
lnn
1
n
1
lnn
1
(Nn
1
n
1
)ln(Nn
1
n
1
).
Differentiating this function by n
1
and n
1
and equating the derivatives to 0 we get:
n
1
= (Nn
1
n
1
) = n
1
n
1
= n
0
= n
1
= N/3
The maximum entropy is
S
max
= kNln3.
Problem 7.5: A system has 3 distinguishable molecules at rest, each with a quantized magnetic
moment which can have z-components +/2 or /2. Find an expression for the distribution
function, f
i
(i denotes the conguration), which maximizes entropy subject to the conditions

i
f
i
= 1,

i
M
i,z
f
i
= ,
where M
i,z
is the magnetic moment of the system in the i-th conguration. For the case = 1/2
compute the entropy and compute f
i
.
Solution 7.5: Let us measure magnetic moment in units . We have 4 congurations with
magnetic moments in parentheses
(3/2), (1/2), (1/2), (3/2).
The congurations with magnetic moments 1/2 are triply degenerated. Denoting the congu-
rations as 1,2, 3 and 4, we construct the functional

S
k
=

i
g
i
f
i
(+M
i,z
ln f
i
).
From this expression we get
+M
i,z
ln f
i
1 = 0 f
i
= exp(1M
i,z
) .
83
Here g
i
is the degeneracy factor: g
1,4
= 1, g
2,3
= 3. We have 2 equations for and :

i
g
i
exp(1M
i,z
) = 1,

i
g
i
M
i,z
exp(1M
i,z
) = .
or
2e
1
_
cosh
3
2
+3cosh

2
_
= 1
2e
1
_
3
2
sinh
3
2
+
3
2
sinh

2
_
= .
For = 1/2 that yields:
cosh
3
2
+3cosh

2
= 3sinh
3
2
+3sinh

2
Denoting x = e
/2
we can rewrite this equation as
x
6
3x
2
4 = 0,
the meaningful solution being
x =

2, or = ln2.
Consequently,
= 1+ln
27

2
4
.
As a result, we have
f
i
=N e
M
i,z
ln2
, N =
2
3/2
27
.
The entropy is
S
k
=

i
g
i
f
i
ln f
i
=

i
g
i
f
i
(lnN +M
i,z
ln2) =lnN
1
2
ln2 = ln
27
4
.
Problem 7.6: A uid in equilibrium is contained in the insulated box of volume V. The uid is
divided (conceptually) into m cells. Compute the variance of the enthalpy uctuations, (H
i
)
2
)
in the i-th cell. (For simplicity assume the uctuations occur at xed particle number, N
i
).
Hint: Use P and S as independent variables.
84 CHAPTER 7. EQUILIBRIUM STATISTICAL MECHANICS
Solution 7.6: We have
H = T S+V P.
Thus
(H)
2
) = T
2
(S)
2
) +V
2
(P)
2
).
From the general theory the uctuation probability is proportional to
w e
W
min
/kT
, W
min
= U T S+PV (7.1)
is the minimum work needed to carry out reversibly the given change in the thermodynamic
quantities of small part considered (relative to which the remainder of the body acts as a medium).
Expanding U in the series, we obtain
W
min
=
1
2
_

2
U
S
2
(S)
2
+2
U
SV
SV +

2
U
V
2
(V)
2
_
=
1
2
_
S
_
U
S
_
V
+V
_
U
V
_
S
_
=
1
2
(ST PV) .
Thus
w exp
_
PV ST
2kT
_
. (7.2)
One can use this general formula and choose different couples as independent variables. In our
case the proper pair is SP. We get
V =
_
V
P
_
S
P+
_
V
S
_
P
S,
T =
_
T
P
_
S
P+
_
T
S
_
P
S
=
_
T
P
_
S
P+
T
C
P
S.
The Maxwell relation following from dH = T dS+V dP is
_
V
S
_
P
=
_
T
P
_
S
.
Thus
V =
_
V
P
_
S
P+
_
T
P
_
S
.
Substituting the above relations into Eq. (7.2) we obtain
w exp
_
1
2kT
_
V
P
_
S
(P)
2

1
2C
P
(S)
2
_
.
85
As a result,
SP) = 0, (S)
2
) = kC
P
, (P)
2
) =kT (P/V)
S
.
Using these relations we get
(H)
2
) = kT
2
C
P
V
2
kT (P/V)
S
.
Problem 7.7: A uid in equilibrium is contained in the insulated box of volume V. The uid
is divided (conceptually) into m cells. Compute the variance of the internal energy uctuations,
(U
i
)
2
) in the i-th cell. (For simplicity assume the uctuations occur at xed particle number,
N
i
). What happens to the internal energy uctuations near the critical point?
Solution 7.7: We have,
U =
_
U
V
_
T
V +
_
U
T
_
V
T =
_
T
_
P
T
_
V
P
_
V +C
V
T .
Now we can use Eq. (7.2) and choose V, T as independent variables. We have:
S =
_
S
T
_
V
T +
_
S
V
_
T
T =
C
V
T
T +
_
P
T
_
V
T ,
P =
_
P
T
_
V
T +
_
P
V
_
T
V .
The we get
w exp
_

C
V
2kT
(T)
2
+
1
2kT
_
P
V
_
T
(V)
2
_
,
T V = 0) (T)
2
) = kT
2
/C
V
, (V)
2
) =kT (V/P)
T
.
Finally,
(U)
2
) =kT
_
T
_
P
T
_
V
P
_
2
_
V
P
_
T
+kT
2
C
V
.
Near the critical point (U)
2
) .
Problem 7.8: What is the partition function for a van der Waals gas with N particles? Note
that the result is phenomenological and might involve some guessing. It is useful to compare it
to the partition function for an ideal gas. Remember that the particles are indistinguishable , so
when using a partition function one must insert a counting factor. Use this partition function to
compute the internal energy, U(N, T,V), the pressure, P(N, T,V), and the entropy, S(N, T,V).
86 CHAPTER 7. EQUILIBRIUM STATISTICAL MECHANICS
Solution 7.8: The partition function is ( (kT)
1
),
Z
N
=
1
N!h
3N

dr
1
. . . dr
N

dp
1
. . . dp
N
e
H(r,p)
.
First we can integrate out the momenta,
Z
(k)
N
=
1
N!h
3N

dp
1
. . . dp
N
e

i
p
2
i
/2m
=
1
N!
3N
T
,
where
T
= h/

2mkT. Since the partition function for the ideal gas is Z


(0)
N
=V
N
Z
(k)
N
Then we
are left with
Z
N
Z
(0)
N
=
1
V
N

dr
1
. . . dr
N
e
V(r
i
)
= 1+
1
V
N

dr
1
. . . dr
N
_
e
V(r
i
)
1
_
1+z
N
.
Let us try to consider rareed gas where only pair interaction is important. Then we can choose
N(N1)/2 pair to get
z
n

N
2
2V
2

dr
1
dr
2
_
e
V(r
1
r
2
)
1
_
.
Introducing the virial coefcient
B(T) =
1
2

dr
_
1e
V(r)
_
,
where r = r
1
r
2
, we obtain
z
N
=N
2
B(T)/V .
We get
A = A
0
kT ln(1+z
n
) A
0
kTz
N
= A
0
+kTN
2
B(T)/V , A
0
kT lnZ
(0)
N
.
In the spirit of the van der Walls model let us assume that the interaction is large at r 2r
0
, and
at r > 2r
0
the interaction is small. Then
B(T) = 2

2r
0
0
r
2
dr
_
1e
V(r)
_
+2


2r
0
r
2
dr
_
1e
V(r)
_
16r
3
0
/3+2


2r
0
r
2
drV(r) ba/kT ,
where b = 16r
3
0
/3 and a =

2r
0
r
2
drV(r). Finally,
z
n
=
N
2
V
_
b
a
kT
_
.
For an ideal gas,
A
0
=NkT ln
eV
N
_
mkT
2 h
2
_
3/2
= N f (T) NkT ln(eV/N).
87
Thus,
A = N f (T) NkT ln(e/N) NkT(lnV Nb/V) N
2
a/V .
Since ln(V Nb) Nb/V +lnV we get
A A
0
NkT ln(1Nb/V) N
2
a/V .
From this we get
P =
A
V
=
NkT
V b

N
2
a
V
, S =
A
T
= S
0
+kNln
_
1
Nb
V
_
,
U =U
0

N
2
a
V
=
3
2
kT
N
2
a
V
.
Problem 7.9: Consider a solid surface to be a two-dimensional lattice with N
s
sites. Assume
that N
a
atoms (N
a
N
s
) are adsorbed on the surface, so that each site has either 0 or 1 adsorbed
atom. At adsorbed atom has energy E =, where > 0. Assume that atoms on the surface do
not interact with each other.
(a) If the surface is at temperature T, compute chemical potential of the adsorbed atoms as a
function of T, , and N
a
/N
s
(use the canonical ensemble).
(b) If the surface is in equilibriumwith an ideal gas of similar atoms at temperature T, compute
the ratio N
a
/N
s
as a function of pressure, P of the gas. Assume the gas has number density
n. (Hint: equate chemical potentials of the adsorbed atoms and the gas).
Solution 7.9:
(a) The canonical partition function is
Z =
1
N
a
!
N
s

i=1
e
E
i
=
N
s
!
N
a
!(N
s
N
a
)!
e
N
a

.
Here = 1/kT. Since N
a
N
s
,
Z
N
N
a
s
N
a
!
e
N
a

.
Equating this expression to e
N
a

we get
= kT ln(N
s
/N
a
).
88 CHAPTER 7. EQUILIBRIUM STATISTICAL MECHANICS
(b) For an ideal gas,
= kT ln
_
P
3
T
kT
_
.
Equating chemical potentials, we obtain


kT
ln
N
s
N
a
= ln
_
P
3
T
kT
_

N
a
N
s
=
P
3
T
kT
e
/kT
.
This expression is valid at high temperatures when
n
3
T
e
/kT
.
Problem 7.10: Consider a two-dimensional lattice lattice lattice in the xy plane with sides of
length L
x
and L
y
which contain N atoms(N is very large) coupled by nearest neighbor harmonic
forces.
(a) Compute the Debye frequency for this lattice.
(b) In the limit T 0, what is the heat capacity?
Solution 7.10: We follow the section 7.E of the book [1]. Since N 1 the boundary conditions
are not important, and we can apply zero boundary conditions. As in this exercise, we use
continual approximation and replace

k
. . .
L
x
L
y
(2)
2

dk
x
dk
y
. . . .
The the density of states is
g() =

k,
(
k,
) =
L
x
L
y
(2)
2

dk (
k,
) .
Here denotes the vibrational mode. In two-dimensional lattice we have two modes, longitudi-
nal, l, and transverse, t. In the Debye approximation it is assumed that the dispersion law
k,
is
linear and isotropic:

k,
= c

k, k
i
=

L
i
s
i
, s
i
= 1, 2, . . . .
where c

is the sound velocity for each mode. Since in two-dimensional case

dk . . . =

kdkd . . . = 2

kdk . . .
we obtain
g() =
L
x
L
y
2 h
2
_
1
c
2
l
+
1
c
2
t
_
.
89
Since the total number of vibrations should be normalized to 2N, where 2 is the number of modes
and N is number of atoms, we should get


D
0
g()d = 2N.
Consequently
2N =
L
x
L
y
4 h
2
_
1
c
2
l
+
1
c
2
t
_

2
D
and
1

D
=

L
x
L
y
8N h
2
_
1
c
2
l
+
1
c
2
t
_
.
The density of states can be expressed as
g() =
4N

2
D
.
As result,
C =
4kN
( h
D
)
2

T
D
/T
0
dx
x
3
e
x
(e
x
1)
2
=
_
2kN, T
24(3)kN(T/T
D
)
2
, T 0.
Here (x) is the Riemann -function, (3) 1.20, T
D
= h
D
/k.
Problem 7.11: A cubic box (with innitely hard walls) of volume V =L
3
contains an ideal gas
of N rigid HCl molecules (assume the effective distance between the H atom and the Cl atom
d = 1.3

A).
(a) If L = 1.0 cm, what is the spacing between the translation energy levels?
(b) Write the partition function for this system (include the translational and rotational contri-
butions). At what temperature do rotational degrees of freedom become important.
(c) Write expressions for the Helmholtz free energy, the entropy, and the heat capacity of
the system for temperatures where the rotational degrees of freedom make a signicant
contributions.
Solution 7.11: Let us put the origin of the reference frame in the cubes corner. Since at the
walls the wave function should vanish, for each component of the momentum p we get:
sin(p

L/ h) = 0 k

= n

/L, k

/ h.
Here n

is an integer number. Thus the energy levels of translational motion are given by the
expression

n
=
0
(n
2
x
+n
2
y
+n
2
z
),
t
=
h
2

2
2mL
2
.
90 CHAPTER 7. EQUILIBRIUM STATISTICAL MECHANICS
Thus the spacing between the levels is

n
i
+1

n
i
= (2n
i
+1)
t
.
The rotational degrees of freedom are determined by the moment if inertia,
I =
m
1
m
2
m
1
+m
2
d
2
,
the energy of rotation being

K
=
r
K(K+1), K = 0, 1, . . . ,
r
=
h
2
2I
.
Thus the spacing between the levels is

K+1

K
= 2(K+1)
r
.
It is important that each rotational level is degenerate by the factor (2K+1).
Thus Z = Z
tr
Z
rot
where
Z
tr
=
_

n
x
=1
e

t
n
2
x
_
3
=
_
V/
3
T
,
t
1,
exp(
t
),
t
1,
Z
rot
=

K=0
(2K+1)e

r
K(K+1)
=
_
kT/
r
,
r
1,
1+3e
2
r
,
r
1.
The following is straightforward.
A = kT lnZ
tr
kT lnZ
rot
;
S =
_
A
T
_
V
, C = T
_
S
T
_
V
=T
_

2
A
T
2
_
V
.
The calculations are obvious.
Problem 7.12: An ideal gas is composed of N red atoms of mass m , N blue atoms of mass
m, and N green atoms of mass m. Atoms of the same color are indistinguishable. Atoms of
different color are distinguishable.
(a) Use the canonical ensemble to compute the entropy of this gas.
(b) Compute the entropy of ideal gas of 3N red atoms of mass m. Does if differ from that of
the mixture. If so, by how much.
91
Solution 7.12: Since the atoms of different color distinguishable the partition function is just
the product of partition function. For each component
Z
N
=
_

k
e
p
2
/2m
_
N
=
1
N!
_
V

3
T
_
N
.
From this expression,
A
N
=NkT ln
eV
N
_
mkT
2 h
2
_
3/2
= N f (T) NkT ln(eV/N).
Here
f (T) =
_
mkT
2 h
2
_
3/2
.
Thus, Z = Z
3
N
, and
A = 3N f (T) 3NkT ln(eV/N), S = 3NKln(eV/N) 3N f
/
(T).
In the case of single-color gas of 3N particles we have
S
1
= 3Nkln(eV/3N) 3N f
/
(T) SS
1
= 3Nln3.
Problem 7.13: An ideal gas consists of a mixture of green and red spin-1/2 particles. All
particles have mass m. A magnetic eld, B, is applied to the system. The green particles have
magnetic moment
G
, and the red particles have magnetic moment
R
, where
R
<
G
. Assume
that temperature is high enough that Fermi statistics can be neglected. The system will be in
equilibrium if chemical potentials the the red and the green gases are equal. Compute the
ratio N
R
/N
G
, where N
R
is the number of red atoms and N
G
is the number of red particles.
Use the canonical ensemble (no other ensembles will be accepted).
Solution 7.13: Let us consider one of the gases since they are noninteracting. We have Z =
Z
tr
Z
s
where Z
tr
is the usual transport partition function while
Z
s
=
_
e
B
+e
B
_
N
= 2
N
cosh
N
(B).
Since
=
kT
N
ln
1
N!
_
V cosh(B)

T
_
N
=kT ln
_
eV cosh(B)
N
T
_
.
Equating chemical potentials for red and green gases we get
cosh(
R
B)
N
R
=
cosh(
G
B)
N
G

N
R
N
G
=
cosh(
R
B)
cosh(
G
B)
.
92 CHAPTER 7. EQUILIBRIUM STATISTICAL MECHANICS
Problem 7.14: Consider a one-dimensional lattice with N lattice sites and assume that i-th
lattice site has spin s
i
=1. The Hamiltonian describing this lattice is
H =
N

i=1
s
i
s
i+1
.
Assume periodic boundary condition, so s
N+1
= s
1
. Compute the correlation function, s
1
s
2
).
How does it behave at very high temperature and at very low temperature?
Solution 7.14: We have,
Z
N
(T) =

s
1
=1
. . .

s
N
=1
exp
_

i
s
i
s
i+1
_
.
As it is recommended in the book [1], we introduce the matrix

P =
_
e

_
= e

I +e

1
.
We have
s
i
[

P[s
i+1
) = e
s
i
s
i+1
.
Then
Z
N
(T) =

s
1
=1
. . .

s
N
=1
s
1
[

P[s
2
)s
2
[

P[s
3
). . . s
N
[

P[s
1
) = Tr

P
N
.
The matrix

P has eigenvalues

1
= 2cosh,
2
= 2sinh.
Thus
Z
N
() = 2
N
(cosh
N
+sinh
N
).
We have
s
i
s
i+1
) =
1
N
lnZ
N

=
cosh
N1
()sinh() +sinh
N1
()cosh()
cosh
N
() +sinh
N
()
.
At low temperature, 1, cosh() sinh() and s
i
s
i+1
) = 1. Thus we have ferromagnet
ordering.
At high temperature,
cosh() 1, sinh() 0 s
i
s
i+1
) = 1.
93
Problem 7.15: In the mean eld approximation to the Ising lattice, the order parameter, s),
satised the equation
s) = tanh
_
s)
T
c
T
_
,
where T
c
=/2k where is the strength of the coupling between lattice sites and is the number
of nearest neighbors.
(a) Show that s) is the following temperature dependence
s)
_
1e
2T
c
/T
, T 0,
_
3(1T/T
c
), T T
c
.
(b) Compute the jump in the heat capacity at T = T
c
.
(c) Compute the magnetic susceptibility, (T, N)
B=0
, in the neighborhood of T
c
both for T >
T
c
and T < T
c
. What is the critical exponent for both cases?
Solution 7.15:
(a) Since tanh 12e
2
at large , we get
s) 12e
2s)T
c
/T
s) 12e
2T
c
/T
.
At small , tanh
3
/3, we obtain
s) s)
T
c
T
+
s)
3
3
_
T
c
T
_
3
.
Thus,
s)
_
3(1T/T
c
).
(b) We will use notations of Sec. 7.F.2 of the book [1]. The the Hamiltonian is
H =
N

i=1
E(, B)s
i
, E(, B) =

2
s) +B. (7.3)
Thus U =NEs) =NEs), and
C =N
_
E(, B)s)
T
_
N,B
=N
_
E +s)
E
s)
__
s)
T
_
N,B
.
At T T
c
0 and B = 0 we get
s) =
_
3(1T/T
c
) C[
T
c
0
=
3
2
N
T
c
.
At T > T
c
the average spin is zero, thus the jump in specic heat is given by C[
T
c
0
.
94 CHAPTER 7. EQUILIBRIUM STATISTICAL MECHANICS
(c) To get magnetic susceptibility we have to differentiate the magnetic moment N = Ns)
with respect to B:
= N
_
s)
B
_
N,B0
.
Dening = (s)/B)
N,B0
we get the following equation at B 0:
=
1
cosh
2
(s)T
c
/T)
_

2
+
_
.
Its solution is
=
2
2cosh
2
(s)T
c
/T) T
c
/T
.
Finally,
=
N
2

T
c
/T
cosh
2
(s)T
c
/T) T
c
/T
.
Above T
c
we have s)=0, and
=
N
2

T
c
T T
c
.
Below T
c
we get
cosh
2
(s)T
c
/T)
_
1+
s)
2
2
_
T
c
T
_
2
_
2
1+3
_
1
T
T
c
_
.
As a result, below T
c
=
N
2

T
c
2(T
c
T)
.
Thus there is a divergence at T T
c
, [T T
c
[
1
, with critical exponent equal to -1.
Problem 7.16: The density of states of an ideal Bose-Einstein gas is
g(E) =
_
E
2
, E > 0,
0, E < 0,
where a is a constant. Compute the temperature for Bose-Einstein condensation.
Solution 7.16: We have the equation for
c
= 1/kT
c
:
N =


E
min
g(E)dE
e

c
E
1
=

3
c


0
x
2
dx
e
x
1
=
2(3)

3
c
.
Here (x) is the Riemann zeta-function. Finally,
kT
c
=
_
N
2(3)
_
1/3
.
95
Problem 7.17: An ideal Bose-Einstein gas consists of noninteracting bosons with mass m
which have an internal degree of freedom which can be described by assuming, that the bo-
son are two-level atoms. Bosons in the ground state have the energy E
0
= p
2
/2m, while bosons
in the excited state have the energy E
1
= p
2
/2m+, where p is the momentum and is the
excitation energy. Assume that kT. Compute the Bose-Einstein condensation temperature,
T
c
, for this gas of two-level bosons. Does the existence of internal degree of the internal degree
of freedom raise or lower the condensation temperature?
Solution 7.17: We start with calculation of density of states.
g
0
(E) =

dp
_
E
p
2
2m
_
=
4
h
3

p
2
dp
_
E
p
2
2m
_
= a

E , a
m
3/2

2
2
h
3
.
In a similar way,
g
1
(E) = a

E .
The equation for
c
= 1/kT
c
is:
N
3/2
c
a
=

xdx
e
x
1
+

_
x
c
dx
e
x
1

(3/2)

2
+

2
e

.
We can solve this equation by iterations since is large. Denoting

0
=
_
a(3/2)

2N
_
2/3
we get

c
=
0
_
1+

a
2N
3/2
0
e

_
2/3

0
+
2

a
6N
_

0
e

.
Thus the internal degree of freedom decreases the condensation temperature.
Problem 7.18: Compute the Clausius-Clapeyron equation for an ideal Bose-Einstein gas and
sketch the coexistence curve. Show that the line of transition points in the Pv plane obeys the
equation
pv
5/3
=
2 h
2
m
g
5/2
(1)
[g
5/2
(1)]
5/3
.
96 CHAPTER 7. EQUILIBRIUM STATISTICAL MECHANICS
Solution 7.18: Let us rst recall the notations. The quantity z = e

is called the fugacity, the


distribution function is written as
n
l
) =
1
e
(
l
)
1
=
z
e

l
z
.
The functions g
i
(z) are introduced as
g
5/2
(z) =
4


0
x
2
dx ln
_
1z
x
2
_
=

=1
z

5/2
, (7.4)
g
3/2
(z) = z
d
dz
g
5/2
(z) =

=1
z

3/2
. (7.5)
Now, the grand canonical potential is
=
V(kT)
5/2
m
3/2

2
2
h
3


0
dy

y ln
_
1ze
y
_
=V
kT

3
T
g
5/2
(z).
Thus the equation of state is
P =
kT

3
T
g
5/2
(z).
In equilibrium with the condensate at T = T
c
,
P
c
=
kT
c

3
T
g
5/2
(1).
The critical particle density in the condensation point is
n
c
) =
g
3/2
(1)

3
T
.
Denoting
T
= a/T
1/2
where a =
_
2 h
2
/mk we get at the critical point:
N
V
=
g
3/2
(1)T
3/2
c
a
3
T
c
=
_
Na
3
Vg
3/2
(1)
_
2/3
.
Thus,
P
c
=
kT
c
g
5/2
(1)

3
T
= ka
2
g
5/2
(1)
[vg
3/2
(1)]
5/3
=
2 h
2
m
g
5/2
(1)
[vg
3/2
(1)]
5/3
.
Problem 7.19: Show that the pressure, P, of an ideal Bose-Einstein gas can be written in the
form P = u, where u is the internal energy per unit volume and a is a constant.
(a) what is u?
(b) What is ?
97
Solution 7.19: Let us do integration by parts in Eq. 7.5 to get
g
5/2
(z) =
2


0
dy

y ln
_
1ze
y
_
=
2

2
3


0
dyy
3/2
z
e
y
z
Calculation the average energy
U =


0
g()d
z
e

z
and recalling that g()

we immediately see that


PV =
2
3
U P =
2
3
u.
Thus, = 2/3.
Problem 7.20: Electrons in a piece of Cu metal can be assumed to behave as an ideal Fermi
gas. Cu metal in the solid state has mass density = 9 gr/cm
3
. Assume that each Cu atom
donates an electron to the Fermi gas. Assume the system is at T = 0 K.
(a) Compute the Fermi energy,
F
, of the electron gas.
(b) Compute the Fermi temperature, T
F
=
F
/k.
Solution 7.20: Let us start with the calculation of the density of states:
g() =
2V 4
h
3

p
2
dp
_

p
2
2m
_
=

2Vm
3/2

2
h
3

Vg
0

.
Here g
0
=

2m
3/2
/
2
h
3
, we have taken into account that spin degeneracy for electrons 2. Inte-
gration expression for densoty of states from 0 to
F
and equationg to the number of electrons
we obtain
(2/3)g
0

3/2
F
= n
e

F
= (3n
e
/2g
0
)
2/3
, .
Here n
e
= N
e
/V = /m
Cu
is the electron density.
98 CHAPTER 7. EQUILIBRIUM STATISTICAL MECHANICS
Problem 7.21: The density of states of an ideal Fermi-Dirac gas is
f (E) =
_
D, if E > 0,
0, if E < 0,
where D is a constant.
(a) Compute the Fermi energy.
(b) Compute the specic heat at low temperature.
Solution 7.21: We have,
N = D


0
dE
e
(E)
+1
=
D

dx
e
x
+1
=
D

ln
_
1+e

_
.
Thus
e

= e
N/D
1 =
1

ln
_
e
N/D
1
_

N
D
kTe
N/DkT
.
The energy is then
U = D


0
E dE
e
(E)
+1
.
Let us use the trick called the Sommerfeld expansion. Consider an auxiliary integral,
I =


0
f (E)dE
e
(E)/kT
+1
=


/kT
f (+kTz)dE
e
z
+1
= kT

/kT
0
f (kTz)dE
e
z
+1
+kT


0
f (+kTz)dE
e
z
+1
=


0
f (E)dE +kT


0
dE
f (+kTz) f (kTz)
e
z
+1
Here we have used the fact that (e
z
+1)
1
= 1(e
z
+1)
1
and that e
/kT
1. Expanding the
integrand, we obtain
I


0
f (E)dE +2(kT)
2
f
/
()


0
zdz
e
z
+1
+. . . =


0
f (E)dE +

2
6
(kT)
2
f
/
() +. . . .
In our case, f (E) = DE, f
/
() = D, and we get
U
D


2
2
+

2
6
(kT)
2
.
The specic heat is then
C
D
=

T
+

2
3
k
2
T .
The rst item is exponentially small at low temperatures, and
C

2
3
Dk
2
T k

2
3
kT

.
99
Problem 7.22: Compute the magnetization of an ideal gas of spin-1/2 fermions in the presence
of magnetic eld. Assume that fermions each have magnetic moment
e
. Find the expression for
magnetization in the limit of weak magnetic eld and T 0 K.
Solution 7.22: We will calculate here only spin (Pauli) susceptibility. Since the spin degree of
freedom leads to additional energy
e
B we get
() =
1
2

0
(
e
B) +
1
2

0
(+
e
B)
where
0
is the grand potential in the absence of magnetic eld,

0
() =
2
3
U =
2A
3


0
E
3/2
dE
e
(E)/kT
+1
, A =

2Vm
3/2

2
h
3
.
As a result,

()
A
=
1
3


0
E
3/2
dE
e
(E
e
B)/kT
+1
+
1
3


0
E
3/2
dE
e
(E+
e
B)/kT
+1
.
In small magnetic elds or at large temperature,
()
0
() +
(
e
B)
2
2

0
()

2
.
Consequently,
M =
_

B
_
T,V,
=
2
e
B

0
()

2
.
Since

=N we get
M =
2
e
B
_
N

_
T,V
.
This formula works in the wide region

e
B .
In very strong magnetic elds the analysis is more complicated. First the chemical potential is
given by the expression
N
A
=
1
3
_
(+
e
B)
3/2
+(
e
B)
3/2
_
.
In a similar way,

()
A
=
2
15
_
(+
e
B)
5/2
+(
e
B)
5/2
_
.
These equations are written assuming that
e
B, otherwise only one itemis present. Physically
it means that only spins-up-states are occupied. Let us consider the case of very strong magnetic
eld,
e
B . Then
() =
2A
15
(+
e
B)
5/2
M =
A
e
3
(+
e
B)
3/2
= N
e
.
100 CHAPTER 7. EQUILIBRIUM STATISTICAL MECHANICS
Problem 7.23: Show that the entropy for an ideal Fermi-Dirac gas (neglecting spin) can be
written in the form
S =k

l
[n
l
)lnn
l
) +(1n
l
))ln(1n
l
))] ,
where n
l
) =
_
e
(
l
)
+1
_
1
.
Solution 7.23: The number of ways to distribute N
l
identical particles among G
l
states with
not more than 1 particle per state is just
W
l
=
G
l
!
N
l
!(G
l
N
l
)!
.
Using the Stirlings formula we obtain
S = k

l
ln[G
l
lnG
l
N
l
lnN
l
(G
l
N
l
)ln(G
l
N
l
)] .
Using the mean occupation number n
l
) = N
l
/G
l
we can rewrite this formula as
S =k

l
G
l
[n
l
)lnn
l
) +(1n
l
))ln(1n
l
))] .
If we ignore spin, then G
l
= 1.
Problem 7.24: To the lowest order in the density, nd the difference in the pressure and the
isothermal compressibility between an ideal boson and ideal fermion gas. Assume that the
fermions and bosons have the same mass and both are spin-less. (Note: You are now consid-
ering fairly high temperature).
Solution 7.24: Assuming that the density of states is given by the expression
g() = g
0
V
1/2
we obtain
=
2
3
g
0
V

3/2
d
e
()
1

2
3
g
0
V

3/2
de
()
_
1e
()
_
.
Introducing the fugacity z = e

we get
P =
2
3
g
0
(kT)
5/2
z

3/2
de

_
1ze

_
=

2
g
0
(kT)
5/2
z
_
1z

2
8
_
.
101
The normalization condition in this case gives
N = g
0
V(kT)
3/2
z

1/2
de

_
1ze

_
=

2
zg
0
V
_
1z

2
4
_
.
As result,
z
0
=
2

g
0
(kT)
3/2
N
V
.
We obtain
P
NkT
V
_
1z
0

2
8
_
_
1z
0

2
4
_
NkT
V
_
1z
0

2
8
_

NkT
V
_
1

2
4

g
0
(kT)
3/2
N
V
_
.
The sign + hold for the Fermi statistics, the sign - holds for the Bose statistics. The com-
pressibility is calculated in a straightforward way since
V
NkT
P
_
1

2
4

g
0
(kT)
5/2
P
_
.
102 CHAPTER 7. EQUILIBRIUM STATISTICAL MECHANICS
Chapter 8
Tests and training
Here we put some exercises for self-testing.
103
104 CHAPTER 8. TESTS AND TRAINING
Problem 8.1:
(a) Discuss the difference between Gibbs and Helmholtz free energy.
(b) Prove the relation
U =T
2
_

T
A
T
_
,
where U is the internal energy.
(c) A body with constant specic heat C
V
is heated under constant volume from T
1
to T
2
. How
much entropy it gains?
(d) Discuss the heating if the same body is in contact with a thermostat at T
2
. In the last case
the heating is irreversible. Show that the total entropy change is positive.
(e) Two similar bodies with temperatures T
1
and T
2
brought into contact. Find the nal tem-
perature and the change in entropy.
Solution 8.1:
(a) Mechanical work under isothermic process is given by
dW = dU dQ = dU T dS = d(U TS).
The function of state
F =U TS
is called Helmholtz free energy. We have
dF = dU T dSSdT = (T dSPdV) T dSSdT =SdT PdV.
Thus,
S =
_
F
T
_
V
, P =
_
F
V
_
T
.
Thus, F is the thermodynamic potential with respect to V and T.
The thermodynamic potential with respect to P and T is called Gibbs free energy. We have
G =U TSPV dG =SdT +V dP.
(b) Substituting
S =
_
F
T
_
V
into denition of F we get the result.
105
(c) We have
S =

T
2
T
1
C
V
T
dT =C
V
ln
_
T
2
T
1
_
.
(d) Use 2nd law of thermodynamics
(e) Energy conservation law yields
C
V
(T
2
T
B
) =C
V
(T
B
T
1
)> T
B
= (T
1
+T
2
)/2.
for the entropy change we have
S =C
V
ln
T
B
T
1
+C
V
ln
T
B
T
2
= 2C
V
ln
_
T
1
+T
2
2

T
1
T
2
_
0.
Problem 8.2: Find uctuation of the energy per particle for the Boltzmann gas of spinless
particles at a given temperature . Assume that
p
= p
2
/2m.
Solution 8.2:
()
2
) =

(
i
))
2
_
=
2
) )
2
.
Now
) =
1
Z
1

i
e

i
=
1
Z
1
Z
1
()

2
) =
1
Z
1

2
i
e

i
=
1
Z
1

2
Z
1
()

2
.
Thus
()
2
) =
1
Z
1

2
Z
1
()

2

_
1
Z
1
Z
1
()

_
2
=

2
logZ
1

2
.
Now we have to specify Z
1
for
i
= p
2
/2m.
Z
1
=

p
exp(
p
) =
1
(2 h)
3


0
4p
2
dpexp(
p
).
It is convenient to introduce the density of states as number of states per energy interval.
g() =

p
(
i
) =
4
(2 h)
3


0
p
2
dp

i
(
i
) =
m
3/2

2
2
h
3

.
106 CHAPTER 8. TESTS AND TRAINING
In this way,
Z
1
=
m
3/2

2
2
h
3


0
d
1/2
exp() =
m
3/2

2
2
h
3

3/2


0
dxx
1/2
exp(x).
As


0
dxx
1/2
exp(x) = (3/2) = (1/2)

we get
Z
1
=
a

3/2
, a =
_

8
m
3/2

2
h
3
logZ
1
= loga
3
2
log.
Thus
()
2
) =
3
2
2
.
Problem 8.3: Two-level systems with random energy spacings are distributed with a constant
probability
p() = P
0
.
Find specic heat of the system.
Solution 8.3: As we know from the lecture,
n
+
n

= e

, n
+
+n

= 1.
Thus
n

=
1
exp() +1
.
The average energy is (we start from the middle of the distance between the levels)
)

=

2
(n
+
n

) =

2
tanh

2
.
The contribution to specic heat is
c() =
)

T
==
2
)

=
1
4
1
cosh
2
(/2)
.
The total specic heat is
C =


0
p()dc() = P
0


0
d
1
4
1
cosh
2
(/2)
=
P
0
2


0
dx
cosh
2
x
=
P
0
2
.
Note that it is proportional to T.
Appendix A
Additional information
A.1 Thermodynamics
A.1.1 Thermodynamic potentials
Thermodynamic potential Notation Independent variables Differential
Internal energy U S, V, N T dSPdV +dN
Heat function (enthalpy) H S, P, N T dS+V dP+dN
Helmholtz free energy F T, V, N SdT PdV +dN
Gibbs free energy G T, P, N SdT +V dP+dN
Landau free energy T, V, SdT PdV Nd
Table A.1: Thermodynamic potentials (summary)
A.1.2 Variable transformation
Jacobian (u, v)/(x, y) is dened as the determinant
(u, v)
(x, y)
=

u/x u/y
v/x v/y

(A.1)
The following relations can be useful:
(v, u)
(x, y)
=
(u, v)
(x, y)
,
(u, y)
(x, y)
=
_
u
x
_
y
,
(u, v)
(x, y)
=
(u, v)
(t, s)

(t, s)
(x, y)
. (A.2)
A.1.3 Derivatives from the equation of state
Important relations arise from the properties of partial differentiations. Consider 3 quantities,
X,Y, Z, related by the equation of state K(X,Y, Z) = const. Now let us consider X,Y as indepen-
107
108 APPENDIX A. ADDITIONAL INFORMATION
dent variables, while Z = Z(X,Y). We have
_
Z
X
_
Y
dX +
_
Z
Y
_
X
dY dZ = 0. (A.3)
If Y, Z are taken as independent variables, then
dX +
_
X
Y
_
Z
dY +
_
X
Z
_
Y
dZ = 0 (A.4)
Now we multiply Eq. A.3 by
_
X
Y
_
Z
and Eq. A.4 by
_
Z
Y
_
X
and subtract the results. We obtain
__
Z
X
_
Y
_
X
Y
_
Z
+
_
Z
Y
_
X
_
dX +
_

_
X
Y
_
Z

_
X
Z
_
Y
_
Z
Y
_
X
_
dZ = 0.
Since dX and dY are independent, this equation is compatible if
_
Z
X
_
Y
_
X
Y
_
Z
+
_
Z
Y
_
X
= 0
_
X
Y
_
Z
+
_
X
Z
_
Y
_
Z
Y
_
X
= 0,
or
_
X
Y
_
Z
_
Y
Z
_
X
_
Z
X
_
Y
=1, (A.5)
_
X
Y
_
Z
=
_
Y
X
_
1
Z
. (A.6)
General scheme for transformation: Consider any quantity F (X,Y), the differential of which
can be expressed as
dF =
_
F
X
_
Y
dX +
_
F
Y
_
X
dX .
Then we divide it by dZ and assume Y = const, (dY = 0). We get
_
F
Z
_
Y
=
_
F
X
_
Y
_
X
Z
_
Y
. (A.7)
Another important relation arises if one divides the expression for dF by xX,
_
F
X
_
Z
=
_
F
X
_
Y
+
_
F
Y
_
X
_
Y
X
_
Z
. (A.8)
Equations (A.7), (A.8) together with Eqs. (A.5), (A.6) and the Maxwell relations are usually
used for transformations and computation of derivatives from the equation of state.
A.2. MAIN DISTRIBUTIONS 109
A.2 Main distributions
Binomial distribution
W
N
(m) =
N!
m!(Nm)!
p
m
(1p)
Nm
. (A.9)
Poisson distribution
P(m) =
m
m
m!
e
m
. (A.10)
Gaussian distribution
G(x) =
1

2
e
(x x)/2
. (A.11)
Distribution of displacemnet for 1D random walk with the step l
G(x) =
1
2l
_
2Np(1p)
e
[x(pq)l]
2
/8Nl
2
p(1p)
. (A.12)
Gaussian integrals
I
1
(a) =

dze
az
2
/2
=
_
2/a. (A.13)

z
2
dze
z
2
/2
= 2
_
dI
1
(a)
da
_
a=1
= I
1
(a)

2. (A.14)
Gaussian distribution for more than one variables Dene entropy as S(x
1
, . . . , x
n
. Then
SS
0
=
1
2

i,k

ik
x
i
x
k
,
ik
=
ki
.
For convenience, let us assume summation over repeated subscripts and rewrite the above equa-
tion
SS
0
=
1
2

ik
x
i
x
k
. (A.15)
Consequently,
w = Ae

1
2

ik
x
i
x
k
.
Let us rst calculate the normalization factor A from

dwdx
1
dx
n
= 1.
To calculate the integral let us introduce the linear transform
x
i
= a
ik
y
k
110 APPENDIX A. ADDITIONAL INFORMATION
to make the quadratic form (refmgs01 diagonal. In order that

ik
x
i
x
k
= y
i
y
i
= y
i
y
k

ik
the relation

ik
a
il
a
km
=
km
(A.16)
should be valid. Denoting determinants of the matrices

and a as and a, respectively, we get
the relation a
2
= 1. The Jacobian of the transformation x
i
y
i
is just a,
J =

(x
1
, . . . , x
n
)
(y
1
, . . . , y
n
)

=[ a[ = a.
Consequently,
Aa
_

dye
y
2
/2
_
n
=
A(2)
n/2
_

= 1.
Thus
w =
_

(2)
n/2
exp
_

1
2

ik
x
i
x
k
_
. (A.17)
Let dene generalized forces as
X
i
=S/x
i
=
ik
x
k
, (A.18)
and rst calculate
x
i
X
k
) =
_

(2)
n/2

x
i

ik
x
k
e

ik
x
i
x
k
/2
dx
1
dx
n
. (A.19)
The easiest way to calculate this integral is to calculate to assume for a while that x
i
,= 0. Then
x =
_

(2)
n/2

x
i
e

ik
(x
i
x
i0
)(x
k
x
k0
/2
dx
1
dx
n
= x
i0
.
Then we can differentiate both sides with respect to x
k0
and then put x
i0
= x
k0
= 0. The l.h.s. is
just x
i
X
k
) while the r.h.s. is
ik
. Thus
x
i
X
k
) =
ik
, or
ik
x
i
x
k
) =
ik
, x
i
x
k
) =
_

1
_
ik
. (A.20)
A.3 The Dirac delta-function
The main property is to single-out one particular value x x
0
of a variable x. It is dened by the
characteristic properties
(x x
0
) =
_
0 for x ,= x
0
for x = x
0
(A.21)
A.4. FOURIER SERIES AND TRANSFORMS 111
but in such way that for any > 0

x
0
+
x
0

(x x
0
)dx = 1. (A.22)
Since delta-function has a very (innitely) sharp peak at x = x
0
and a unit area

B
A
f (x)(x x
0
)dx =
_
f (x
0
) if A < x
0
< B
0 otherwise
. (A.23)
Representations for the Dirac -function
Let introduce a positive parameter and at nal stage tend it to zero. Physically it means that
is less than all other involved scales. The main representations are the following: rectangular,
(x) =
_

1
for /2 < x < /e
0 otherwise
, (A.24)
Lorentzian,
(x) =
1

x
2
+
2
, (A.25)
Gaussian,
(x) =
1

2
e
x
2
/2
2
, (A.26)
and integral,
(x) =
1
2

e
ikx
dk (A.27)
We will also need a representation for the Kronekkers symbol

n,0
=
1
2

e
n
d. (A.28)
A.4 Fourier Series and Transforms
Fourier series
Generally the Fourier series is dened as an expansion of a real function in series as
f (x) =
a
0
2
+

k=1
(a
k
sinkx +b
k
coskx) . (A.29)
The sum of the series is periodic with the period 2.
Another form is complex Fourier series
f (x) =

k=
c
k
e
ikx
, c
k
= (a
k
b
k
)/2, c
0
= a
0
/2. (A.30)
112 APPENDIX A. ADDITIONAL INFORMATION
Since

2
0
sinmxsinnxdx =
_

mn
, m ,= 0,
0, m = 0,

2
0
cosmxcosnxdx =
_

mn
, m ,= 0,
2, m = n = 0,

2
0
sinmxcosnxdx =
_
0, for integer m, n
we have
_
_
a
k
b
k
c
k
_
_
=
1

2
0
dt f (t)
_
_
coskt
sinkt
1
2
e
ikt
_
_
, k ,= 0. (A.31)
At k = 0,
a
0
= c
0
=
1
2

2
0
dt f (t).
If the function has a discontinuity at x = x
0
then the series gives
f (x
0
) =
1
2
[ f (x
0
+0) + f (x
0
0)] .
If one is willing to change the interval 2 2L than k has to be replaced by k/L and the
normalization factor is 1/L rather than 1/.
Advantages of the Fourier series: it can represent discontinuous functions. Let us take an
example of periodic function with the period 2 dened as
F(x) =
_
1, /2 < x < /2
0, < x </2, /2 < x < ,
This function is even, thus
b
k
= 0, a
k
= (2/k)sink/2.
Below we plot the sum of 5 and 50 Fourier harmonics. It is useful to know the Parsevals identity
1

[ f (x)]
2
dx =
a
2
0
2
+

k=1
(a
2
k
+b
2
k
).
Fourier transform
We assume L and replace

k=
(. . .) L

dk(. . .).
g(k) =
1
2

f (t)e
ikt
dt . (A.32)
A.4. FOURIER SERIES AND TRANSFORMS 113
0
0.2
0.4
0.6
0.8
1
1 0.8 0.6 0.4 0.2 0.2 0.4 0.6 0.8 1
x
Figure A.1:
The completeness condition
f (x) =
1
2

dke
ikt

dx f (x)e
ikx
. (A.33)
The Dirac delta-function is represented as
(x t) =
1
2

dke
ik(tx)
. (A.34)
We have several useful rules for the Fourier transform. If g(k) is the Fourier transform of f (x)
then
f
/
(x) d f /dx ikg(k);
f
(n)
(x) d
n
f /dx
n
(ik)
n
g(k);

dy f
1
(y) f
2
(x y) g
1
(k) g
2
(k) (convolution theorem) ;

f
1
(t) f
2
(t)dt

g
1
(k) f
2
(k)dk;

f
1
(t) f

2
(t)dt

g
1
(k) f

2
(k)dk the Parsevals theorem. (A.35)
These properties are very important for solving differential equations, etc.
114 APPENDIX A. ADDITIONAL INFORMATION
f (x) =
1

F()e
ix
dx F() =
1

f (x)e
ix
dx
1
a
2
+x
2
, a > 0
_

2
e
a[[
a
f (x) =
_
1, [x[ < a
0, [x[ > a
_
2

sin(a)

e
a
2
x
2
1
a

2
e

2
/4a
Table A.2: Table of Fourier transform pairs
Examples
One-dimensional diffusion: The equation has the form
n(x, t)
t
=D

2
n(x, t)
x
2
.
Let us assume that we add a particle at time t = 0 at the point x = 0. Then the equation together
with initial condition can be written as
n(x, t)
t
+D

2
n(x, t)
x
2
= (x)(t).
The Fourier transform of this equation has the form:
(Dk
2
i)n(k, ) = 1 n(k, ) =
1
(2)
2
1
Dk
2
i
.
Now we can come back to real space-time representation:
n(x, t) =

dk
2

d
2
1
Dk
2
i
=

dk
2
e
Dk
2
[t[
=
1
2
_
D[t[
e
x
2
/4D[t[
.
Appendix B
Maple Printouts
Quick access: 1, 2.1, 2.2, 2.17, 3.7, 3.12, 3.13, 4.9, 4.10, 4.13, 4.14, 5.1, 5.2, 5.3, 5.4, 5.5, 5.7,
5.8
Test in mathematics
>
F:=(xi,eta)->xi^4+xi^3+eta*xi^2:
>
F1:=(xi,eta)->subs(y=xi,diff(F(y,eta),y)): solve(F1(xi,eta)=0,xi);
0,
3
8
+
1
8

932,
3
8

1
8

932
One extremum at eta >9/32, otherwise 3 extrema
>
x1:=-3/8+1/8*sqrt(9-32*eta): x2:=-3/8-1/8*sqrt(9-32*eta):
>
F2:=(xi,eta)->subs(y=xi,diff(F1(y,eta),y)):
>
g0:=eta->simplify(F2(0,eta)): g1:=eta->simplify(F2(x1,eta)):
>
g2:=eta->simplify(F2(x2,eta)):
>
g0(eta); g1(eta); g2(eta);
2
9
8

3
8

9324
9
8
+
3
8

9324
>
plot(g0(eta),g1(eta),g2(eta),eta=-0.1..9/32);
115
116 APPENDIX B. MAPLE PRINTOUTS
0.5
1
1.5
2
2.5
0.1 0.05 0.05 0.1 0.15 0.2 0.25
eta
x0-> maximum at eta <0, otherwise minimum; x1-> minimum at eta>0, maximum at
9/32>eta>0; x2-> minimum at 9/32>eta.
Two specic values: eta=0 and eta=9/32.
>
plot(F(xi,0), xi=-1.1..0.45); plot(F(xi,9/32), xi=-0.7..0.25);
117
0.1
0.05
0.05
0.1
1 0.8 0.6 0.4 0.2 0.2 0.4
xi
0.005
0.01
0.015
0.02
0.025
0.03
0.035
0.6 0.4 0.2 0.2
xi
Problem 1.2
>
f1:=x->ln((1-x)/(1+x)): plot(f1(x),x=-1..1);
118 APPENDIX B. MAPLE PRINTOUTS
6
4
2
0
2
4
6
1 0.8 0.6 0.4 0.2 0.2 0.4 0.6 0.8 1
x
>
f2:=x->ln(abs(tan(Pi*x))): plot(f2(x),x=0..1,f2=-5..5);
4
2
0
2
4
f2
0.2 0.4 0.6 0.8 1
x
119
>
simplify(exp(4*ln(x))-(x^2+1)^2+2*x^2+1);
0
Problem 1.3
>
simplify((sin(x))^(-2)-(tan(x))^(-2)-1,trig);
0
>
assume(alpha>0): assume(beta>0):
>
k:=alpha->sum(exp((I*beta-alpha)*n),n=0..infinity):
>
a1:=alpha->evalc(Re(k(alpha))): a2:=alpha->evalc(Im(k(alpha))):
>
a1(alpha); a2(alpha);

(cos() e

)
(cos() e

)
2
+sin()
2
e

sin()
(cos() e

)
2
+sin()
2
Problem 1.4
>
assume(zeta>0): assume(n,natural):
>
simplify(int(x^n*exp(-zeta*x),x=0..infinity));

(n1)
(n +1)
>
int((x^2+a^2)^(-1),x); int((x^2-a^2)^(-1),x);
arctan(
x
a
)
a
1
2
ln(x a)
a

1
2
ln(a+x)
a
>
int(exp(-zeta*x^2/2),x=0..infinity);
1
2

Problem 2.1
(a)
>
uax:=(x,y)->-y/(x^2+y^2): uay:=(x,y)->x/(x^2+y^2):
>
simplify(diff(uax(x,y),y)-diff(uay(x,y),x));
0
The differential is exact
>
ua:=(x,y)->int(uax(xi,y),xi=0..x): ua(x,y);
arctan(
x
y
)
120 APPENDIX B. MAPLE PRINTOUTS
(b)
>
ubx:=(x,y)->(y-x^2): uby:=(x,y)->(x+y^2):
>
simplify(diff(ubx(x,y),y)-diff(uby(x,y),x));
0
The differential is exact
>
ub:=(x,y)->int(ubx(xi,y),xi=0..x)+int(uby(x,eta),eta=0..y)-int(int(di
>
ff(ubx(xi,eta),eta),eta=0..y),xi=0..x);
ub := (x, y)

x
0
ubx(, y)d+

y
0
uby(x, )d

x
0

y
0

ubx(, )dd
>
ub(x,y);
yx
1
3
x
3
+
1
3
y
3
(c)
>
ucx:=(x,y)->(2*y^2-3*x): ucy:=(x,y)->-4*x*y:
>
simplify(diff(ucx(x,y),y)-diff(ucy(x,y),x));
8y
The differential is not exact
Problem 2.2
General denitions
>
ua:=(x,y)->int(ux(xi,b),xi=a..x)+int(uy(x,eta),eta=b..y);
ua := (x, y)

x
a
ux(, b)d+

y
b
uy(x, )d
>
ub:=(x,y)->int(uy(a,eta),eta=b..y)+int(ux(xi,y),xi=a..x);
ub := (x, y)

y
b
uy(a, )d+

x
a
ux(, y)d
>
ux1:=(x,y)->2*x*y+x^2: uy1:=(x,y)->x^2:
>
ua1:=(x,y)->int(ux1(xi,b),xi=a..x)+int(uy1(x,eta),eta=b..y):
>
ub1:=(x,y)->int(uy1(a,eta),eta=b..y)+int(ux1(xi,y),xi=a..x):
>
ua1(x,y); ub1(x,y); simplify(ua1(x,y)-ub1(x,y));
b(x
2
a
2
) +
1
3
x
3

1
3
a
3
+x
2
(y b)
a
2
(y b) +y(x
2
a
2
) +
1
3
x
3

1
3
a
3
0
>
ux2:=(x,y)->y*(2-2*y): uy2:=(x,y)->-x^2:
121
>
ua2:=(x,y)->int(ux2(xi,b),xi=a..x)+int(uy2(x,eta),eta=b..y):
>
ub2:=(x,y)->int(uy2(a,eta),eta=b..y)+int(ux2(xi,y),xi=a..x):
>
ua2(x,y); ub2(x,y); simplify(ua2(x,y)-ub2(x,y));
b(22b)(x a) x
2
(y b)
a
2
(y b) +y(22y)(x a)
2bx 2ba2b
2
x +2b
2
ax
2
y +x
2
b+a
2
y a
2
b2yx +2ya+2y
2
x 2y
2
a
Why?
>
simplify(diff(ux2(x,y),y)-uy2(x,y),x);
24y +x
2
The differential is not exact.
Problem 2.17
>
P:=(T,v)->R*T/(v-b)-alpha/v^2:
>
num:=(diff(P(T,v),T))^2: den:=diff(P(T,v),v);
den :=
RT
(v b)
2
+
2
v
3
>
simplify(-T*num/den);

v
3
R
2
T
RT v
3
+2v
2
4vb+2b
2
>
factor(2*alpha*v^2-4*alpha*v*b+2*alpha*b^2);
2(v +b)
2
Problem 3.7
>
P:=(v,T)->(8*T/(3*v-1))-3/v^2;
P := (v, T) 8
T
3v 1

3
v
2
>
plot(P(v,1.5),P(v,1),P(v,0.5)
>
,v=0.5..1.2,thickness=3,color=black);
122 APPENDIX B. MAPLE PRINTOUTS
4
2
0
2
4
6
8
10
12
0.6 0.7 0.8 0.9 1 1.1 1.2
v
For T=0.5 there is no stable region at all. The phse transition can exist only nes T=1.
>
plot(P(v,1.05),P(v,1),P(v,0.85)
>
,v=0.45..5.3,P=0..2,thickness=3,color=black);
0
0.2
0.4
0.6
0.8
1
1.2
1.4
1.6
1.8
2
P
1 2 3 4 5
v
>
?plot,color
Problem 3.12
>
phi:=xi->(A/3)*(xi+eta)^2-(2*B/27)*(xi+eta)^3+(C/9)*(xi+eta)^4:
>
series(phi(xi),xi=0,5);
(
1
3
A
2

2
27
B
3
+
1
9
C
4
) +(
2
3
A
2
9
B
2
+
4
9
C
3
)+(
1
3
A
2
9
B+
2
3
C
2
)
2
+
(
2
27
B+
4
9
C)
3
+
1
9
C
4
>
c3:=eta->-2/27*B+4/9*C*eta: solve(c3(eta)=0,eta);
1
6
B
C
>
c1:=(A,eta)->2/3*A*eta-2/9*B*eta^2+4/9*C*eta^3;
c1 := (A, )
2
3
A
2
9
B
2
+
4
9
C
3
>
solve(c1(A,B/(6*C)),A);
1
27
B
2
C
>
c2:=subs(eta=B/(6*C),A=B^2/(27*C),1/3*A-2/9*B*eta+2/3*C*eta^2);
c2 :=
1
162
B
2
C
>
delta:=xi->(C/9)*xi^4+c2*xi^2:
>
delta1:=xik->subs(y=xi,diff(delta(y),y)): delta1(xi);
4
9
C
3

1
81
B
2

C
>
solve(4/9*C*xi^2-1/81*B^2/C,xi);

1
6
B
C
,
1
6
B
C
Problem 3.13
>
P:=(v,T)->8*T/(3*v-1)-3/(T*v^2);
P := (v, T) 8
T
3v 1

3
T v
2
>
p1:=(nu,epsilon)=P(1+nu,1+epsilon)-1;
p1 := (, ) = 8
1+
2+3

3
(1+)(1+)
2
1
>
aux:=series(8*(1+epsilon)/(2+3*nu)-3/(1+epsilon)/(1+nu)^2-1,nu=0,4):a
>
ux;
(3+4
3
1+
) +(66 +
6
1+
)+(9
1
1+
+9+9)
2
+(12
1
1+

27
2

27
2
)

3
+O(
4
)
>
aux1:=(epsilon)->3+4*epsilon-3/(1+epsilon)+(-6-6*epsilon+6/(1+epsilon
>
))*nu+(-9*1/(1+epsilon)+9+9*epsilon)*nu^2
>
+(12*1/(1+epsilon)-27/2-27/2*epsilon)*nu^3;
aux1 := 3+4
3
1+
+(66 +
6
1+
)+(9
1
1+
+9+9)
2
+(12
1
1+

27
2

27
2
)
3
>
aux2:=series(aux1(epsilon),epsilon=0,2);
aux2 :=
3
2

3
+(7
51
2

3
+18
2
12) +O(
2
)
Simplied equation of state
>
p:=(nu,epsilon)->-3/2*nu^3+(7-12*nu)*epsilon;
p := (, )
3
2

3
+(712)
Critical point - epsilon=0;
>
d1:=(nu,epsilon)->subs(y=nu,diff(p(y,epsilon),y)): d1(nu,epsilon);

9
2

2
12
The curve is symmetric. That leads to vg=-vl=v
>
solve(p(v,epsilon)= p(-v,epsilon),v);;
0, 2

2, 2

2
Critical exponent beta=1/2
Along the coexitence curve, v2=-8epsilon. exponent gamma=1
Function of pressure
>
P1:=(rho,T)->8*T*rho/(3-rho)-3*rho^2/T:
Near the critical point
>
P1(1+xi,1);
8
1+
2
3(1+)
2
>
series(8*(1+xi)/(2-xi)-3*(1+xi)^2,xi=0);
1+
3
2

3
+
3
4

4
+
3
8

5
+O(
6
)
the exponent delta=3
Problem 4.9
>
assume(z>0): int(1/sqrt(z-x^2),x=0..sqrt(z));
1
2

>
f:=k->1/(1-2*I*k): d1:=k->subs(y=k,diff(f(y),y)):
>
d2:=k->subs(y=k,diff(d1(y),y)): d3:=k->subs(y=k,diff(d2(y),y)):
>
d1(0); d2(0); d3(0);
2I
8
48I
Problem 4.10
>
P:=(N,k)->(N!/(k!*(N-k)!))*(1/4)^k*(3/4)^(N-k):
>
evalf(P(12,3));
.2581036091
>
evalf(P(120,30));
.08385171464
>
G:=(N,k)->(1/sqrt(3*N*Pi/8))*exp(-(k-N/4)^2*8/(3*N));
G := (N, k)
e
(8/3
(k1/4N)
2
N
)
_
3
8
N
>
evalf(G(12,3));
.2659615201
>
evalf(G(120,30));
.08410441740
>
plot(P(12,k),G(12,k),k=0..12);
0
0.05
0.1
0.15
0.2
0.25
2 4 6 8 10 12
k
>
plot(P(120,k),G(120,k),k=0..120);
0
0.02
0.04
0.06
0.08
20 40 60 80 100 120
k
Problem 4.13
beta = sigma2
>
assume(beta>0): assume(a,real): assume(N>0):
>
P:=k->(2*Pi*beta)^(-1/2)*int(exp(-(x-a)^2/(2*beta)-I*k*x),x=-infinity.
>
.infinity): simplify(P(k));
e
(1/2I k(2a+I k))
>
P1:=s->(2*Pi)^(-1)*int(exp(I*(s-N*a)*k-k^2*N*beta/2),k=-infinity..inf
>
inity): P1(s);
1
2
e
(1/2
(s+N a)
2
N
)

_
N
>
int(s*P1(s),s=-infinity..infinity);
N a
>
int(s^2*P1(s),s=-infinity..infinity)-N^2*a^2;
N
Problem 4.14
>
assume(a>0): assume(d>a): assume(N>0):
>
simplify((2*a)^(-1)*int(x^2,x=d-a..d+a));
d
2
+
1
3
a
2
>
simplify(N*(d^2+1/3*a^2)+N*(N-1)*d^2-N^2*d^2);
1
3
N a
2
Problem 5.1
>
with(linalg):
>
Q:=array([[0,1,0],[1/8,1/2,3/8],[0,1/2,1/2]]);
Q :=
_

_
0 1 0
1
8
1
2
3
8
0
1
2
1
2
_

_
>
QT:=transpose(Q):
>
eigenvectors(Q);
[1, 1, [1, 1, 1]], [
1
4
, 1,
_
6,
3
2
, 1
_
], [
1
4
, 1, [4, 1, 2]]
>
eigenvectors(QT);
[
1
4
, 1, [1, 2, 3]], [
1
4
, 1, [1, 2, 1]], [1, 1, [1, 8, 6]]
>
r1:=vector([1, 1, 1]); r2:=vector([6, -3/2, 1]); r3:=vector([4, 1,
>
-2]);
r1 := [1, 1, 1]
r2 :=
_
6,
3
2
, 1
_
r3 := [4, 1, 2]
>
l1:=vector([1, 8, 6]); l2:=vector([1, -2, 1]); l3:=vector([1, 2,
>
-3]);
l1 := [1, 8, 6]
l2 := [1, 2, 1]
l3 := [1, 2, 3]
>
n1:=15: n2:=10: n3:=12:
>
P1:=array([[l1[1]*r1[1], l1[2]*r1[1],l1[3]*r1[1]], [l1[1]*r1[2],
>
l1[2]*r1[2],l1[3]*r1[2]],[l1[1]*r1[3], l1[2]*r1[3],l1[3]*r1[3]]]);
P1 :=
_
_
1 8 6
1 8 6
1 8 6
_
_
>
P2:=array([[l2[1]*r2[1], l2[2]*r2[1],l2[3]*r2[1]], [l2[1]*r2[2],
>
l2[2]*r2[2],l2[3]*r2[2]],[l2[1]*r2[3], l2[2]*r2[3],l2[3]*r2[3]]]);
P2 :=
_

_
6 12 6
3
2
3
3
2
1 2 1
_

_
>
P3:=array([[l3[1]*r3[1], l3[2]*r3[1],l3[3]*r3[1]], [l3[1]*r3[2],
>
l3[2]*r3[2],l3[3]*r3[2]],[l3[1]*r3[3], l3[2]*r3[3],l3[3]*r3[3]]]);
P3 :=
_
_
4 8 12
1 2 3
2 4 6
_
_
>
P:=s->(1/n1)*P1+(1/n2)*(-1/4)^s*P2+(1/n3)*(1/4)^s*P3;
>
Question (a)
P := s
P1
n1
+
(
1
4
)
s
P2
n2
+
(
1
4
)
s
P3
n3
>
Pr:=s->evalm(P(s)): Pr(s); Pr(0);
_

_
1
15
+
3
5
(
1
4
)
s
+
1
3
(
1
4
)
s
8
15

6
5
(
1
4
)
s
+
2
3
(
1
4
)
s
2
5
+
3
5
(
1
4
)
s
(
1
4
)
s
1
15

3
20
(
1
4
)
s
+
1
12
(
1
4
)
s
8
15
+
3
10
(
1
4
)
s
+
1
6
(
1
4
)
s
2
5

3
20
(
1
4
)
s

1
4
(
1
4
)
s
1
15
+
1
10
(
1
4
)
s

1
6
(
1
4
)
s
8
15

1
5
(
1
4
)
s

1
3
(
1
4
)
s
2
5
+
1
10
(
1
4
)
s
+
1
2
(
1
4
)
s
_

_
_
_
1 0 0
0 1 0
0 0 1
_
_
>
p0:=vector([0,1,0]);
p0 := [0, 1, 0]
>
PS:=s->evalm(p0&*P(s)): PS(s);
Question (b)
_
1
15

3
20
(
1
4
)
s
+
1
12
(
1
4
)
s
,
8
15
+
3
10
(
1
4
)
s
+
1
6
(
1
4
)
s
,
2
5

3
20
(
1
4
)
s

1
4
(
1
4
)
s
_
>
PS(2); PS(infinity);
>
Answer: after 2 steps 3/8, after many steps 2/5
_
1
16
,
9
16
,
3
8
_
_
1
15
,
8
15
,
2
5
_
>
av:=s->1*PS(s)[1]+4*PS(s)[2]+9*PS(s)[3]: av(s);
>
Moment
29
5

3
10
(
1
4
)
s

3
2
(
1
4
)
s
>
av:=sum(m^2*PS(s)[m],m=1..3);
29
5

3
10
(
1
4
)
s

3
2
(
1
4
)
s
>
corr:=s->simplify(sum(n^2*sum(m^2*PS(s)[m]*Pr(s)[m,n],
>
m=1..3),n=1..3)): corr(s);
79
50
(1)
(1+s)
4
(s)

33
4
16
(s)
+
81
100
(1)
(1+2s)
16
(s)
+
11
2
(1)
(1+s)
16
(s)

3
2
4
(s)
+
841
25
>
corr(0);
16
>
corr(infinity);
841
25
Problem 5.2
>
with(linalg):
Warning, the protected names norm and trace have been redefined and
unprotected
>
Q:=array([[0,1/2,1/2],[0,0,1],[3/4,1/4,0]]);
Q :=
_

_
0
1
2
1
2
0 0 1
3
4
1
4
0
_

_
>
QT:=transpose(Q):
>
r:=eigenvectors(Q):
>
l:=eigenvectors(QT):
>
r[1][1]; r[2][1]; r[3][1];
1

1
2
+
1
4
I

1
2

1
4
I

2
>
l[1][1]; l[2][1]; l[3][1];
1

1
2
+
1
4
I

1
2

1
4
I

2
>
r[1][3]; r[2][3]; r[3][3];
[1, 1, 1]

1
6

1
3
I

2, 1,
1
2
+
1
4
I

2
_

1
6
+
1
3
I

2, 1,
1
2

1
4
I

2
_

>
l[1][3]; l[2][3]; l[3][3];

_
6
5
, 1,
8
5
_

_
1,
1
3

1
3
I

2,
2
3
+
1
3
I

2
_

_
1,
1
3
+
1
3
I

2,
2
3

1
3
I

2
_

>
r1:=vector([1, 1, 1]): r2:=vector([-1/6-1/3*I*sqrt(2), 1,
>
-1/2+1/4*I*sqrt(2)]): r3:=vector([-1/6+1/3*I*sqrt(2), 1,
>
-1/2-1/4*I*sqrt(2)]): l1:=vector([6/5, 1, 8/5]): l2:=vector([1,
>
-1/3-1/3*I*sqrt(2), -2/3+1/3*I*sqrt(2)]): l3:=vector([1,
>
-1/3+1/3*I*sqrt(2), -2/3-1/3*I*sqrt(2)]):
>
n1:=evalm(l1&*r1): n2:=evalc(evalm(l2&*r2)):
>
n3:=evalc(evalm(l3&*r3)):
Probability matrices.
>
P1:=array([[l1[1]*r1[1], l1[2]*r1[1],l1[3]*r1[1]], [l1[1]*r1[2],
>
l1[2]*r1[2],l1[3]*r1[2]],[l1[1]*r1[3], l1[2]*r1[3],l1[3]*r1[3]]]);
P1 :=
_

_
6
5
1
8
5
6
5
1
8
5
6
5
1
8
5
_

_
>
P2:=array([[l2[1]*r2[1], l2[2]*r2[1],l2[3]*r2[1]], [l2[1]*r2[2],
>
l2[2]*r2[2],l2[3]*r2[2]],[l2[1]*r2[3], l2[2]*r2[3],l2[3]*r2[3]]]);
P2 :=
_

1
6

1
3
I

2 (
1
3

1
3
I

2)(
1
6

1
3
I

2) (
2
3
+
1
3
I

2)(
1
6

1
3
I

2)
1
1
3

1
3
I

2
2
3
+
1
3
I

1
2
+
1
4
I

2 (
1
3

1
3
I

2)(
1
2
+
1
4
I

2) (
2
3
+
1
3
I

2)(
1
2
+
1
4
I

2)
_

_
>
P3:=array([[l3[1]*r3[1], l3[2]*r3[1],l3[3]*r3[1]], [l3[1]*r3[2],
>
l3[2]*r3[2],l3[3]*r3[2]],[l3[1]*r3[3], l3[2]*r3[3],l3[3]*r3[3]]]);
P3 :=
_

1
6
+
1
3
I

2 (
1
3
+
1
3
I

2)(
1
6
+
1
3
I

2) (
2
3

1
3
I

2)(
1
6
+
1
3
I

2)
1
1
3
+
1
3
I

2
2
3

1
3
I

1
2

1
4
I

2 (
1
3
+
1
3
I

2)(
1
2

1
4
I

2) (
2
3

1
3
I

2)(
1
2

1
4
I

2)
_

_
>
P:=s->(1/n1)*P1+(1/n2)*(r[2][1])^s*P2+(1/n3)*(r[3][1])^s*P3;
>
Question (a)
P := s
P1
n1
+
r
21
s
P2
n2
+
r
31
s
P3
n3
>
Pr:=s->evalm(P(s)):
>
p0:=vector([1,0,0]);
p0 := [1, 0, 0]
>
PS:=s->evalm(p0&*P(s)): PS(s)[1];
6
19
+
(
1
2
+
1
4
I

2)
s
(
1
6

1
3
I

2)

1
3
I

2
+
(
1
2

1
4
I

2)
s
(
1
6
+
1
3
I

2)

1
3
+I

2
>
evalc(PS(infinity)[1]); evalc(PS(2)[1]);
6
19
3
8
Problem 5.3
>
with(linalg):
Warning, the protected names norm and trace have been redefined and
unprotected
>
Q:=array([[0,1/2,1/2],[1/3,0,2/3],[1/3,2/3,0]]);
Q :=
_

_
0
1
2
1
2
1
3
0
2
3
1
3
2
3
0
_

_
>
QT:=transpose(Q):
>
r:=eigenvectors(Q):
>
l:=eigenvectors(QT):
>
r[1][1]; r[2][1]; r[3][1];
1
3
2
3
1
>
l[1][1]; l[2][1]; l[3][1];
1
3
1
2
3
>
r[3][3]; r[1][3]; r[2][3];
[1, 1, 1]
[3, 1, 1]
[0, 1, 1]
>
l[2][3]; l[1][3]; l[3][3];

_
1,
3
2
,
3
2
_

[2, 1, 1]
[0, 1, 1]
>
r1:=vector([1, 1, 1]): r2:=vector([-3, 1, 1]): r3:=vector([0, -1,
>
1]): l1:=vector([1, 3/2, 3/2]): l2:=vector([-2, 1, 1]): l3:=vector([0,
>
-1, 1]):
>
n1:=evalm(l1&*r1): n2:=evalc(evalm(l2&*r2)):
>
n3:=evalc(evalm(l3&*r3)):
Probability matrices.
>
P1:=array([[l1[1]*r1[1], l1[2]*r1[1],l1[3]*r1[1]], [l1[1]*r1[2],
>
l1[2]*r1[2],l1[3]*r1[2]],[l1[1]*r1[3], l1[2]*r1[3],l1[3]*r1[3]]]);
P1 :=
_

_
1
3
2
3
2
1
3
2
3
2
1
3
2
3
2
_

_
>
P2:=array([[l2[1]*r2[1], l2[2]*r2[1],l2[3]*r2[1]], [l2[1]*r2[2],
>
l2[2]*r2[2],l2[3]*r2[2]],[l2[1]*r2[3], l2[2]*r2[3],l2[3]*r2[3]]]);
P2 :=
_
_
6 3 3
2 1 1
2 1 1
_
_
>
P3:=array([[l3[1]*r3[1], l3[2]*r3[1],l3[3]*r3[1]], [l3[1]*r3[2],
>
l3[2]*r3[2],l3[3]*r3[2]],[l3[1]*r3[3], l3[2]*r3[3],l3[3]*r3[3]]]);
P3 :=
_
_
0 0 0
0 1 1
0 1 1
_
_
>
P:=s->(1/n1)*P1+(1/n2)*(r[3][1])^s*P2+(1/n3)*(r[2][1])^s*P3;
>
Question (a)
P := s
P1
n1
+
r
31
s
P2
n2
+
r
21
s
P3
n3
>
Pr:=s->evalm(P(s)): Pr(s);
>
Conditional probability
_

_
1 0 0
0
1
2
+
1
2
(
2
3
)
s
1
2

1
2
(
2
3
)
s
0
1
2

1
2
(
2
3
)
s
1
2
+
1
2
(
2
3
)
s
_

_
>
p0:=vector([0,0,1]):
>
PS:=s->evalm(p0&*P(s)): PS(s);
Probability vector
_
1
4

1
4
(
2
3
)
s
,
1
8
+
1
8
(
2
3
)
s
,
7
8
+
1
8
(
2
3
)
s
_
>
PS(infinity); PS(0);
_
0,
1
2
,
1
2
_
[0, 0, 1]
>
moment:=1*PS(s)[1]+2*PS(s)[2]+3*PS(s)[3]: moment;
5
2
+
1
2
(
2
3
)
s
>
corr:=s->simplify(sum(n*sum(m*PS(s)[m]*Pr(s)[m,n], m=1..3),n=1..3)):
>
corr(s);
25
4
+
5
4
3
(2s)
(1)
(2s)
2
(2s)
+(1)
s
3
(1s)
2
(1+s)
>
corr(0); corr(infinity);
9
25
4
Problem 5.4
>
with(linalg):
Warning, the protected names norm and trace have been redefined and
unprotected
>
Q0:=s->array([[0,(cos(Pi*s/2))^2,(sin(Pi*s/2))^2],[1/4
>
+1/2*(sin(Pi*s/2))^2,0,1/4+1/2*(cos(Pi*s/2))^2],[1/2*(cos(Pi*s/2))^2,1
>
/2+1/2*(sin(Pi*s/2))^2,0]]): Q0(s);
_

_
0 cos(
1
2
s)
2
sin(
1
2
s)
2
1
4
+
1
2
sin(
1
2
s)
2
0
1
4
+
1
2
cos(
1
2
s)
2
1
2
cos(
1
2
s)
2
1
2
+
1
2
sin(
1
2
s)
2
0
_

_
>
simplify(Q0(2*s));
_

_
0 cos(s)
2
1cos(s)
2
3
4

1
2
cos(s)
2
0
1
4
+
1
2
cos(s)
2
1
2
cos(s)
2
1
1
2
cos(s)
2
0
_

_
Since [cos(Pi*s)]2=1, we get
>
Q:=array([[0,1,0],[1/4,0,3/4],[1/2,1/2,0]]);
Q :=
_

_
0 1 0
1
4
0
3
4
1
2
1
2
0
_

_
>
QT:=transpose(Q):
>
r:=eigenvectors(Q):
>
l:=eigenvectors(QT):
>
r[1][1]; r[2][1]; r[3][1];
1

1
2
+
1
4
I

1
2

1
4
I

2
>
l[1][1]; l[2][1]; l[3][1];

1
2
+
1
4
I

1
2

1
4
I

2
1
>
r[3][3]; r[1][3]; r[2][3];

_
1,
1
2

1
4
I

2,
1
6
+
1
3
I

2
_

[1, 1, 1]

_
1,
1
2
+
1
4
I

2,
1
6

1
3
I

2
_

>
l[3][3]; l[1][3]; l[2][3];

_
1,
8
5
,
6
5
_

1
3

1
3
I

2,
2
3
+
1
3
I

2, 1
_

1
3
+
1
3
I

2,
2
3

1
3
I

2, 1
_

>
r1:=vector([1, 1, 1]): r2:=vector([1, -1/2+1/4*I*sqrt(2),
>
-1/6-1/3*I*sqrt(2)]): r3:=vector([1, -1/2-1/4*I*sqrt(2),
>
-1/6+1/3*I*sqrt(2)]): l1:=vector([1, 8/5, 6/5]):
>
l2:=vector([-1/3-1/3*I*sqrt(2), -2/3+1/3*I*sqrt(2), 1]):
>
l3:=vector([-1/3+1/3*I*sqrt(2), -2/3-1/3*I*sqrt(2), 1]):
>
n1:=evalm(l1&*r1): n2:=evalc(evalm(l2&*r2)):
>
n3:=evalc(evalm(l3&*r3)):
Probability matrices.
>
P1:=array([[l1[1]*r1[1], l1[2]*r1[1],l1[3]*r1[1]], [l1[1]*r1[2],
>
l1[2]*r1[2],l1[3]*r1[2]],[l1[1]*r1[3], l1[2]*r1[3],l1[3]*r1[3]]]);
P1 :=
_

_
1
8
5
6
5
1
8
5
6
5
1
8
5
6
5
_

_
>
P2:=array([[l2[1]*r2[1], l2[2]*r2[1],l2[3]*r2[1]], [l2[1]*r2[2],
>
l2[2]*r2[2],l2[3]*r2[2]],[l2[1]*r2[3], l2[2]*r2[3],l2[3]*r2[3]]]);
P2 :=
_

1
3

1
3
I

2
2
3
+
1
3
I

2 1
(
1
3

1
3
I

2)(
1
2
+
1
4
I

2) (
2
3
+
1
3
I

2)(
1
2
+
1
4
I

2)
1
2
+
1
4
I

2
(
1
3

1
3
I

2)(
1
6

1
3
I

2) (
2
3
+
1
3
I

2)(
1
6

1
3
I

2)
1
6

1
3
I

2
_

_
>
P3:=array([[l3[1]*r3[1], l3[2]*r3[1],l3[3]*r3[1]], [l3[1]*r3[2],
>
l3[2]*r3[2],l3[3]*r3[2]],[l3[1]*r3[3], l3[2]*r3[3],l3[3]*r3[3]]]);
P3 :=
_

1
3
+
1
3
I

2
2
3

1
3
I

2 1
(
1
3
+
1
3
I

2)(
1
2

1
4
I

2) (
2
3

1
3
I

2)(
1
2

1
4
I

2)
1
2

1
4
I

2
(
1
3
+
1
3
I

2)(
1
6
+
1
3
I

2) (
2
3

1
3
I

2)(
1
6
+
1
3
I

2)
1
6
+
1
3
I

2
_

_
>
P:=s->(1/n1)*P1+(1/n2)*(r[1][1])^(2*s)*P2+(1/n3)*(r[2][1])^(2*s)*P3;
>
Question (a)
P := s
P1
n1
+
r
11
(2s)
P2
n2
+
r
21
(2s)
P3
n3
>
Pr:=s->evalm(P(s)):
>
Conditional probability
>
p01:=vector([1,0,0]): p02:=vector([0,1,0]):
>
PS1:=s->evalm(p01&*P(s)): PS2:=s->evalm(p02&*P(s)):
>
PS1(s)[1];
5
19
+

1
3

1
3
I

1
3
I

2
+
(
1
2
+
1
4
I

2)
(2s)
(
1
3
+
1
3
I

2)

1
3
+I

2
>
PS1(s)[2];
8
19
+

2
3
+
1
3
I

1
3
I

2
+
(
1
2
+
1
4
I

2)
(2s)
(
2
3

1
3
I

2)

1
3
+I

2
>
PS2(s)[1];
5
19
+
(
1
3

1
3
I

2)(
1
2
+
1
4
I

2)

1
3
I

2
+
(
1
2
+
1
4
I

2)
(2s)
(
1
3
+
1
3
I

2)(
1
2

1
4
I

2)

1
3
+I

2
>
PS2(s)[2];
8
19
+
(
2
3
+
1
3
I

2)(
1
2
+
1
4
I

2)

1
3
I

2
+
(
1
2
+
1
4
I

2)
(2s)
(
2
3

1
3
I

2)(
1
2

1
4
I

2)

1
3
+I

2
The results can be transformed to polar form by the command polar.
Problem 5.5
>
assume(N,natural): assume(k,real): assume(s>0); assume(n,integer):
>
P:=(n,s,N)->(1/(2*N+1))*(1+2*sum(cos(2*Pi*k*n/(2*N+1))*exp(-2*s*(sin(
>
Pi*k/(2*N+1)))^2),k=1..N)):
>
P(n,s,N);
1+2
_
N

k=1
cos(2
k n
2N +1
)e
(2s sin(
k
2N+1
)
2
)
_
2N +1
>
Pm:=[seq(P(i,2,2),i=-2..2)]: evalf(Pm);
[.1220644066, .2223516301, .3111679263, .2223516301, .1220644066]
Problem 5.8
>
with(linalg):
Warning, the protected names norm and trace have been redefined and
unprotected
>
w:=matrix(3,3,[0,1/2,1/2,1/3,0,1/3,1/3,1/3,0]);
w :=
_

_
0
1
2
1
2
1
3
0
1
3
1
3
1
3
0
_

_
>
M:=matrix(3,3,[-1,1/2,1/2,1/3,-1,1/3,1/3,1/3,-1]);
M :=
_

_
1
1
2
1
2
1
3
1
1
3
1
3
1
3
1
_

_
>
MT:=transpose(M):
>
eigenvectors(M);
[
5
6
+
1
6

13, 1,
_

1
2
+
1
2

13, 1, 1
_
], [
5
6

1
6

13, 1,
_

1
2

1
2

13, 1, 1
_
],
[
4
3
, 1, [0, 1, 1]]
>
eigenvectors(MT);
[
4
3
, 1, [0, 1, 1]], [
5
6
+
1
6

13, 1,
_

1
3
+
1
3

13, 1, 1
_
],
[
5
6

1
6

13, 1,
_

1
3

1
3

13, 1, 1
_
]
>
lambda:=vector([-5/6+1/6*sqrt(13),-5/6-1/6*sqrt(13),-4/3]):
>
r1:=vector([-1/2+1/2*sqrt(13), 1, 1]); r2:=vector([-1/2-1/2*sqrt(13),
>
1, 1]); r3:=vector([0, -1, 1]);
r1 :=
_

1
2
+
1
2

13, 1, 1
_
r2 :=
_

1
2

1
2

13, 1, 1
_
r3 := [0, 1, 1]
>
l1:=vector([-1/3+1/3*sqrt(13), 1, 1]); l2:=vector([-1/3-1/3*sqrt(13),
>
1, 1]); l3:=vector([0, -1, 1]);
l1 :=
_

1
3
+
1
3

13, 1, 1
_
l2 :=
_

1
3

1
3

13, 1, 1
_
l3 := [0, 1, 1]
1
>
n1:=evalm(l1&*r1): n2:=evalm(l2&*r2): n3:=evalm(l3&*r3):
>
Q1:=-sum((1/n1)*r1[1]*l1[k]/lambda[1],k=1..3):
>
Q2:=-sum((1/n2)*r1[1]*l1[k]/lambda[2],k=1..3):
>
Q3:=-sum((1/n3)*r1[1]*l1[k]/lambda[1],k=1..3):
>
evalf(Q1+Q2+Q3);
>
First passage time
13.64536110
>
:
Bibliography
[1] L. E. Reichl A modern course in statistical physics, 1998.
141

You might also like